0% found this document useful (0 votes)
24 views95 pages

Urology Mcqs

Uploaded by

Shehroz Qureshi
Copyright
© © All Rights Reserved
We take content rights seriously. If you suspect this is your content, claim it here.
Available Formats
Download as PDF, TXT or read online on Scribd
0% found this document useful (0 votes)
24 views95 pages

Urology Mcqs

Uploaded by

Shehroz Qureshi
Copyright
© © All Rights Reserved
We take content rights seriously. If you suspect this is your content, claim it here.
Available Formats
Download as PDF, TXT or read online on Scribd
You are on page 1/ 95

Urology Topics BCQs

Here are 30 Best Choice Questions (BCQs) related to urolithiasis, based on urology ward
practice and Bailey & Love's Short Practice of Surgery.

Causes of Urolithiasis
1. Which of the following is the most common cause of calcium oxalate stones?

A) Hyperuricosuria

B) Hypercalciuria

C) Hyperoxaluria

D) Hypocitraturia

Answer: B) Hypercalciuria

2. Which dietary factor is most associated with the formation of uric acid stones?

A) High-protein diet

B) Low-calcium diet

C) High-sodium diet

D) High-fiber diet

Answer: A) High-protein diet

3. Which metabolic disorder is most commonly linked to the formation of struvite stones?

A) Hyperparathyroidism

B) Cystinuria

C) Hyperuricemia

D) Urinary tract infections

Answer: D) Urinary tract infections

4. Which of the following is NOT a recognized risk factor for urolithiasis?

A) Low fluid intake

B) Obesity

Printed with ChatGPT to PDF 1/95


C) Hypoparathyroidism

D) Hyperparathyroidism

Answer: C) Hypoparathyroidism

5. Which of the following medications is associated with the risk of stone formation?

A) Aspirin

B) Furosemide

C) Allopurinol

D) Spironolactone

Answer: B) Furosemide

Types of Urolithiasis
6. The most common type of kidney stones in patients is composed of:

A) Uric acid

B) Calcium oxalate

C) Struvite

D) Cystine

Answer: B) Calcium oxalate

7. Which of the following types of stones are radiolucent on X-ray?

A) Calcium oxalate stones

B) Struvite stones

C) Uric acid stones

D) Calcium phosphate stones

Answer: C) Uric acid stones

8. Which of the following stones are classically seen in patients with recurrent urinary tract
infections?

A) Calcium oxalate stones

B) Struvite stones

Printed with ChatGPT to PDF 2/95


C) Cystine stones

D) Uric acid stones

Answer: B) Struvite stones

9. Cystine stones are most commonly associated with which of the following conditions?

A) Hypercalcemia

B) Hyperoxaluria

C) Genetic disorder

D) Hyperuricosuria

Answer: C) Genetic disorder

10. Which of the following statements about staghorn calculi is true?

A) They are always composed of calcium oxalate.

B) They are frequently associated with alkaline urine.

C) They are radiolucent on imaging.

D) They are most commonly composed of uric acid.

Answer: B) They are frequently associated with alkaline urine.

Complications of Urolithiasis
11. Which of the following is the most common complication of kidney stones?

A) Hydronephrosis

B) Renal cell carcinoma

C) Acute kidney injury

D) Hypertension

Answer: A) Hydronephrosis

12. Chronic obstruction from urolithiasis may lead to which of the following conditions?

A) Pyelonephritis

B) Hypertension

C) Renal atrophy

Printed with ChatGPT to PDF 3/95


D) All of the above

Answer: D) All of the above

13. Which of the following is a potential complication of untreated staghorn calculi?

A) Urosepsis

B) Nephrocalcinosis

C) Renal cell carcinoma

D) Bladder cancer

Answer: A) Urosepsis

14. Urolithiasis with infection can lead to:

A) Pyelonephritis

B) Pyonephrosis

C) Urosepsis

D) All of the above

Answer: D) All of the above

15. What is the most likely complication if a stone obstructs the ureteropelvic junction?

A) Hydronephrosis

B) Renal vein thrombosis

C) Renal cell carcinoma

D) Nephrotic syndrome

Answer: A) Hydronephrosis

Management of Urolithiasis
16. What is the first-line imaging modality for suspected renal colic?

A) X-ray KUB

B) Ultrasound

C) CT scan (non-contrast)

D) MRI

Printed with ChatGPT to PDF 4/95


Answer: C) CT scan (non-contrast)

17. A 6mm kidney stone is found in the mid-ureter. What is the most appropriate initial
management?

A) Observation and hydration

B) Ureteroscopy

C) Extracorporeal shock wave lithotripsy (ESWL)

D) Open surgery

Answer: C) Extracorporeal shock wave lithotripsy (ESWL)

18. For patients with uric acid stones, which of the following treatments is preferred?

A) Potassium citrate

B) Allopurinol

C) Calcium supplements

D) Thiazide diuretics

Answer: A) Potassium citrate

19. Which of the following medications is used in the medical management of hypercalciuria
to reduce stone formation?

A) Spironolactone

B) Hydrochlorothiazide

C) Allopurinol

D) Furosemide

Answer: B) Hydrochlorothiazide

20. Which stone size is most likely to require surgical intervention?

A) Less than 5mm

B) Between 5-10mm

C) Between 10-20mm

D) More than 20mm

Printed with ChatGPT to PDF 5/95


Answer: D) More than 20mm

21. What is the definitive treatment for a patient with recurrent staghorn calculi and poor
renal function?

A) Percutaneous nephrolithotomy (PCNL)

B) Ureteroscopy

C) Extracorporeal shock wave lithotripsy (ESWL)

D) Nephrectomy

Answer: D) Nephrectomy

22. Which of the following is indicated for stones in a pregnant patient?

A) CT scan

B) Percutaneous nephrolithotomy

C) Ultrasound-guided management

D) Open surgery

Answer: C) Ultrasound-guided management

23. The management of choice for a 3mm distal ureteral stone with minimal symptoms is:

A) Watchful waiting

B) Ureteroscopy

C) ESWL

D) Percutaneous nephrolithotomy

Answer: A) Watchful waiting

24. For patients with struvite stones, the treatment approach involves:

A) Antibiotic therapy and stone removal

B) Dietary modification

C) Increased fluid intake alone

D) Only antibiotic therapy

Answer: A) Antibiotic therapy and stone removal

Printed with ChatGPT to PDF 6/95


25. What is the first-line pharmacological treatment for cystine stones?

A) Potassium citrate

B) Penicillamine

C) Allopurinol

D) Hydrochlorothiazide

Answer: B) Penicillamine

26. Which of the following procedures is most appropriate for large renal calculi (>2 cm)?

A) Ureteroscopy

B) Percutaneous nephrolithotomy (PCNL)

C) Extracorporeal shock wave lithotripsy (ESWL)

D) Open surgery

Answer: B) Percutaneous nephrolithotomy (PCNL)

27. What is the best non-surgical management for uric acid stones?

A) Acidification of urine

B) Alkalinization of urine

C) Thiazide diuretics

D) Restriction of oxalates

Answer: B) Alkalinization of urine

Urology Ward Care


28. A patient with recurrent calcium oxalate stones should be advised to:

A) Reduce fluid intake

B) Increase calcium intake

C) Reduce sodium intake

D) Increase oxalate-rich foods

Answer: C) Reduce sodium intake

Printed with ChatGPT to PDF 7/95


29. The initial management of a patient presenting to the emergency department with acute
renal colic should be:

A) Intravenous antibiotics

B) Immediate CT scan

C) Intravenous fluids and analgesia

D) Surgical intervention

Answer: C) Intravenous fluids and analgesia

30. In cases of obstructive urolithiasis with infection, the priority of treatment is:

A) Immediate stone removal

B) Antibiotic therapy and urgent decompression

C) Watchful waiting

D) Lithotripsy

Answer: B) Antibiotic therapy and urgent decompression

Here are 30 Best Choice Questions (BCQs) on Benign Prostatic Hyperplasia (BPH) focusing on
its causes, complications, diagnosis, and management based on Bailey & Love's Short Practice
of Surgery and urology ward practice.

Causes of BPH
1. Which of the following factors is most associated with the development of benign prostatic
hyperplasia (BPH)?

A) Age

B) Smoking

C) Alcohol consumption

D) Sexual activity

Answer: A) Age

2. The primary hormonal factor involved in the development of BPH is:

A) Testosterone

B) Estrogen

Printed with ChatGPT to PDF 8/95


C) Dihydrotestosterone (DHT)

D) Progesterone

Answer: C) Dihydrotestosterone (DHT)

3. Which part of the prostate is most commonly affected by hyperplasia in BPH?

A) Central zone

B) Transitional zone

C) Peripheral zone

D) Anterior fibromuscular stroma

Answer: B) Transitional zone

4. BPH results primarily from the proliferation of:

A) Glandular epithelium only

B) Stromal and glandular tissues

C) Smooth muscle cells only

D) Fibrous tissues

Answer: B) Stromal and glandular tissues

5. Which of the following lifestyle factors is least associated with the risk of developing BPH?

A) Obesity

B) Sedentary lifestyle

C) Alcohol consumption

D) Low vegetable intake

Answer: C) Alcohol consumption

Complications of BPH
6. Which of the following is a common complication of untreated BPH?

A) Prostate cancer

B) Acute urinary retention

C) Erectile dysfunction

Printed with ChatGPT to PDF 9/95


D) Urethral stricture

Answer: B) Acute urinary retention

7. Chronic obstruction due to BPH can lead to which of the following renal complications?

A) Glomerulonephritis

B) Hydronephrosis

C) Pyelonephritis

D) Renal cell carcinoma

Answer: B) Hydronephrosis

8. Which of the following is NOT a potential complication of BPH?

A) Recurrent urinary tract infections

B) Bladder stones

C) Hematuria

D) Testicular cancer

Answer: D) Testicular cancer

9. BPH can lead to overflow incontinence due to:

A) Urethral sphincter damage

B) Bladder overactivity

C) Bladder outlet obstruction

D) Neurological impairment

Answer: C) Bladder outlet obstruction

10. Which of the following is the most common cause of gross hematuria in patients with
BPH?

A) Urethral trauma

B) Bladder cancer

C) Prostatic hypervascularity

D) Urethral stricture

Printed with ChatGPT to PDF 10/95


Answer: C) Prostatic hypervascularity

Diagnosis of BPH
11. What is the most commonly used questionnaire to assess the severity of symptoms in
patients with BPH?

A) International Prostate Symptom Score (IPSS)

B) American Urological Association Symptom Index (AUA-SI)

C) WHO Symptom Score

D) SF-36 Quality of Life Questionnaire

Answer: A) International Prostate Symptom Score (IPSS)

12. On digital rectal examination (DRE), which of the following findings is suggestive of BPH?

A) Asymmetry of the prostate

B) Nodularity of the prostate

C) Smooth, firm, and enlarged prostate

D) Hard, irregular prostate

Answer: C) Smooth, firm, and enlarged prostate

13. Which investigation is the gold standard for confirming bladder outlet obstruction in
BPH?

A) Uroflowmetry

B) Post-void residual urine volume

C) Urodynamic studies

D) Cystoscopy

Answer: C) Urodynamic studies

14. In a patient with suspected BPH, which initial investigation should be performed to
evaluate the prostate size?

A) CT scan

B) MRI

Printed with ChatGPT to PDF 11/95


C) Transrectal ultrasound (TRUS)

D) Uroflowmetry

Answer: C) Transrectal ultrasound (TRUS)

15. What is the significance of measuring prostate-specific antigen (PSA) in patients with
BPH?

A) To differentiate BPH from prostate cancer

B) To monitor disease progression

C) To guide antibiotic therapy

D) PSA is not useful in BPH

Answer: A) To differentiate BPH from prostate cancer

Management of BPH
16. Which of the following is the first-line pharmacological treatment for BPH?

A) 5-alpha-reductase inhibitors

B) Alpha-blockers

C) Anticholinergics

D) PDE-5 inhibitors

Answer: B) Alpha-blockers

17. Which of the following is a common side effect of alpha-blockers used to treat BPH?

A) Erectile dysfunction

B) Orthostatic hypotension

C) Gynecomastia

D) Blurred vision

Answer: B) Orthostatic hypotension

18. 5-alpha-reductase inhibitors, such as finasteride, work by:

A) Relaxing smooth muscle in the prostate

B) Reducing dihydrotestosterone levels

Printed with ChatGPT to PDF 12/95


C) Increasing bladder muscle contraction

D) Enhancing urethral sphincter tone

Answer: B) Reducing dihydrotestosterone levels

19. Which of the following surgical interventions is considered the gold standard for BPH?

A) Open prostatectomy

B) Transurethral resection of the prostate (TURP)

C) Laser ablation

D) Urolift procedure

Answer: B) Transurethral resection of the prostate (TURP)

20. In patients with moderate to severe symptoms of BPH, which combination therapy is
often recommended?

A) Alpha-blocker and anticholinergic

B) Alpha-blocker and 5-alpha-reductase inhibitor

C) PDE-5 inhibitor and alpha-blocker

D) 5-alpha-reductase inhibitor and PDE-5 inhibitor

Answer: B) Alpha-blocker and 5-alpha-reductase inhibitor

21. Minimally invasive therapies like transurethral microwave therapy (TUMT) are typically
reserved for:

A) Men with small prostates

B) Men who are not surgical candidates

C) Men with prostate cancer

D) Men with acute urinary retention

Answer: B) Men who are not surgical candidates

22. Which of the following is a potential complication of TURP?

A) Erectile dysfunction

B) Urethral stricture

Printed with ChatGPT to PDF 13/95


C) Retrograde ejaculation

D) All of the above

Answer: D) All of the above

23. A patient presents with acute urinary retention secondary to BPH. The most appropriate
immediate management is:

A) Start alpha-blocker therapy

B) TURP

C) Catheterization

D) Urethral dilatation

Answer: C) Catheterization

24. What is the recommended follow-up interval for a patient with BPH on medical therapy?

A) Every 6 months

B) Every 12 months

C) Every 2 years

D) No follow-up needed unless symptoms worsen

Answer: A) Every 6 months

25. In a patient with BPH and predominantly irritative symptoms, which class of drugs might
be added to the treatment regimen?

A) Anticholinergics

B) 5-alpha-reductase inhibitors

C) PDE-5 inhibitors

D) Antibiotics

Answer: A) Anticholinergics

26. Which of the following is an indication for surgical management in BPH?

A) Mild symptoms with minimal bother

B) Acute urinary retention

Printed with ChatGPT to PDF 14/95


C) Nocturia only

D) Mild decrease in urinary flow

Answer: B) Acute urinary retention

27. For patients who cannot undergo TURP due to comorbidities, which of the following is a
viable alternative?

A) Open prostatectomy

B) Urolift procedure

C) Transurethral incision of the prostate (TUIP)

D) Laser vaporization

Answer: D) Laser vaporization

Urology Ward Care


28. In the urology ward, a common post-operative complication of TURP is:

A) TUR syndrome

B) Prostatic abscess

C) Urinary tract infection

D) Bowel perforation

Answer: A) TUR syndrome

29. A 75-year-old male presents with worsening lower urinary tract symptoms despite
maximal medical therapy. The next step in management should be:

A) Change medication

B) Watchful waiting

C) Cystoscopy and consideration of surgery

D) Uroflowmetry only

Answer: C) Cystoscopy and consideration of surgery

**

Printed with ChatGPT to PDF 15/95


Here are 30 Best Choice Questions (BCQs) on prostate cancer, covering causes,
complications, diagnosis, staging, and management based on Bailey & Love's Short Practice
of Surgery and urology ward practice.

Causes of Prostate Cancer


1. Which of the following is the strongest risk factor for prostate cancer?

A) Smoking

B) Family history

C) Obesity

D) Alcohol consumption

Answer: B) Family history

2. Prostate cancer is most commonly associated with which of the following genetic
mutations?

A) BRCA1/BRCA2 mutations

B) APC mutations

C) RET mutations

D) KRAS mutations

Answer: A) BRCA1/BRCA2 mutations

3. The primary hormonal driver of prostate cancer growth is:

A) Estrogen

B) Dihydrotestosterone (DHT)

C) Progesterone

D) Luteinizing hormone

Answer: B) Dihydrotestosterone (DHT)

4. Which racial/ethnic group has the highest risk of developing prostate cancer?

A) Caucasian

B) African American

Printed with ChatGPT to PDF 16/95


C) Hispanic

D) Asian

Answer: B) African American

5. Which of the following is a modifiable risk factor for prostate cancer?

A) Age

B) Family history

C) Diet high in saturated fats

D) Ethnicity

Answer: C) Diet high in saturated fats

Complications of Prostate Cancer


6. The most common site of metastasis in prostate cancer is:

A) Liver

B) Bone

C) Lungs

D) Brain

Answer: B) Bone

7. Which of the following complications is most commonly associated with advanced


prostate cancer?

A) Erectile dysfunction

B) Hypercalcemia

C) Spinal cord compression

D) Bladder cancer

Answer: C) Spinal cord compression

8. Prostate cancer commonly metastasizes to the spine, leading to which of the following
complications?

A) Pathological fractures

Printed with ChatGPT to PDF 17/95


B) Radiculopathy

C) Paraplegia

D) All of the above

Answer: D) All of the above

9. Which of the following conditions can result from untreated advanced prostate cancer?

A) Renal failure

B) Benign prostatic hyperplasia

C) Testicular cancer

D) Urinary tract infections

Answer: A) Renal failure

10. Paraneoplastic syndromes associated with prostate cancer may include:

A) Hypercalcemia

B) Syndrome of inappropriate antidiuretic hormone (SIADH)

C) Cushing’s syndrome

D) Lambert-Eaton myasthenic syndrome

Answer: A) Hypercalcemia

Diagnosis of Prostate Cancer


11. The initial screening test for prostate cancer is:

A) Digital rectal examination (DRE)

B) Prostate-specific antigen (PSA) test

C) MRI of the pelvis

D) Transrectal ultrasound (TRUS)

Answer: B) Prostate-specific antigen (PSA) test

12. On digital rectal examination, which finding is most concerning for prostate cancer?

A) Smooth, symmetrically enlarged prostate

B) Hard, irregular nodules on the prostate

Printed with ChatGPT to PDF 18/95


C) Soft, fluctuant prostate

D) Firm but regular prostate

Answer: B) Hard, irregular nodules on the prostate

13. Which imaging modality is most useful for evaluating the extent of prostate cancer and
detecting metastases?

A) Transrectal ultrasound

B) CT scan of the abdomen and pelvis

C) Multiparametric MRI

D) PET scan

Answer: C) Multiparametric MRI

14. A definitive diagnosis of prostate cancer is made by:

A) PSA level

B) Digital rectal exam

C) Prostate biopsy

D) MRI findings

Answer: C) Prostate biopsy

15. Which of the following PSA levels is most concerning for prostate cancer in a man over 60
years old?

A) PSA < 1 ng/mL

B) PSA between 1–2.5 ng/mL

C) PSA between 2.5–4 ng/mL

D) PSA > 10 ng/mL

Answer: D) PSA > 10 ng/mL

Staging of Prostate Cancer


16. In the TNM staging system for prostate cancer, T2 refers to:

A) Tumor confined to the prostate

Printed with ChatGPT to PDF 19/95


B) Tumor invades beyond the prostate capsule

C) Tumor invades adjacent structures

D) Tumor involves regional lymph nodes

Answer: A) Tumor confined to the prostate

17. Which of the following best describes stage T3b in prostate cancer staging?

A) Tumor invades seminal vesicles

B) Tumor confined to the prostate

C) Tumor invades bladder neck

D) Tumor involves pelvic lymph nodes

Answer: A) Tumor invades seminal vesicles

18. The Gleason score in prostate cancer is used to:

A) Predict response to chemotherapy

B) Estimate prognosis based on histological grading

C) Determine metastasis potential

D) Stage the extent of disease spread

Answer: B) Estimate prognosis based on histological grading

19. A Gleason score of 8-10 indicates:

A) Low-grade cancer

B) Intermediate-grade cancer

C) High-grade cancer

D) No cancer present

Answer: C) High-grade cancer

20. Which test is most useful for detecting bone metastases in advanced prostate cancer?

A) CT scan

B) Bone scan

C) MRI of the spine

Printed with ChatGPT to PDF 20/95


D) PET scan

Answer: B) Bone scan

Management of Prostate Cancer


21. Active surveillance is most appropriate for which group of prostate cancer patients?

A) High-grade, aggressive prostate cancer

B) Elderly patients with low-risk, localized prostate cancer

C) Patients with metastatic prostate cancer

D) Patients with symptomatic prostate cancer

Answer: B) Elderly patients with low-risk, localized prostate cancer

22. The main goal of radical prostatectomy is:

A) Preservation of sexual function

B) Complete removal of the prostate to cure cancer

C) Pain relief

D) Control of urinary symptoms

Answer: B) Complete removal of the prostate to cure cancer

23. Which of the following is a potential side effect of radical prostatectomy?

A) Urinary incontinence

B) Erectile dysfunction

C) Bladder neck contracture

D) All of the above

Answer: D) All of the above

24. In patients with metastatic prostate cancer, the mainstay of treatment is:

A) Surgery

B) Chemotherapy

C) Hormonal therapy (androgen deprivation therapy)

D) Radiation therapy

Printed with ChatGPT to PDF 21/95


Answer: C) Hormonal therapy (androgen deprivation therapy)

25. Which of the following medications is commonly used for androgen deprivation therapy
(ADT) in prostate cancer?

A) Leuprolide

B) Cisplatin

C) Finasteride

D) Tamoxifen

Answer: A) Leuprolide

26. Which of the following is a typical side effect of androgen deprivation therapy?

A) Gynecomastia

B) Anemia

C) Decreased bone density

D) All of the above

Answer: D) All of the above

27. In localized prostate cancer, external beam radiation therapy is often used as an
alternative to:

A) Chemotherapy

B) Hormonal therapy

C) Radical prostatectomy

D) Palliative care

Answer: C) Radical prostatectomy

28. A patient with prostate cancer presents with spinal cord compression due to bone
metastasis. What is the most appropriate immediate management?

A) Surgical decompression

B) High-dose steroids

C) Chemotherapy

Printed with ChatGPT to PDF 22/95


D) Radiation therapy

Answer: B) High-dose steroids

Urology Ward Care


29. In patients with advanced prostate cancer, which of the following treatments is used to
manage bone metastases?

A) Bisphosphonates or denosumab

B) Androgen deprivation therapy

C) External beam radiation

D) Radical prostatectomy

Answer: A) Bisphosphonates or denosumab

30. The most appropriate follow-up for a patient after radical prostatectomy includes
monitoring:

A) PSA levels

B) Urodynamic studies

C) Bone scans every year

D) Repeat biopsies every 6 months

Answer: A) PSA levels

These questions cover various aspects of prostate cancer from its pathogenesis to advanced
management, useful for learning in a clinical or academic setting.

Here are 30 Best Choice Questions (BCQs) on urinary tract infections (UTIs), covering causes,
symptoms, types, diagnosis, and management, based on Bailey & Love's Short Practice of
Surgery and urology ward practice.

Causes of UTIs
1. What is the most common causative organism for community-acquired urinary tract
infections (UTIs)?

A) Staphylococcus aureus

B) Escherichia coli

Printed with ChatGPT to PDF 23/95


C) Klebsiella pneumoniae

D) Pseudomonas aeruginosa

Answer: B) Escherichia coli

2. Which of the following conditions increases the risk of developing a UTI?

A) Chronic constipation

B) Gallstones

C) Diabetes mellitus

D) Hypertension

Answer: C) Diabetes mellitus

3. In hospitalized patients, which of the following is a common risk factor for developing
catheter-associated urinary tract infections (CAUTIs)?

A) Prolonged catheterization

B) Short-term catheterization

C) Use of clean intermittent catheterization

D) Antibiotic prophylaxis

Answer: A) Prolonged catheterization

4. Which of the following factors predisposes women to a higher risk of UTIs compared to
men?

A) Long urethra

B) Increased vaginal pH

C) Shorter urethra

D) Higher testosterone levels

Answer: C) Shorter urethra

5. Which of the following congenital abnormalities is associated with recurrent UTIs?

A) Hypospadias

B) Vesicoureteral reflux

Printed with ChatGPT to PDF 24/95


C) Undescended testes

D) Phimosis

Answer: B) Vesicoureteral reflux

Symptoms of UTIs
6. A typical symptom of a lower urinary tract infection (cystitis) is:

A) Flank pain

B) Dysuria

C) Fever and chills

D) Hematuria with clots

Answer: B) Dysuria

7. What is a common symptom of pyelonephritis, a type of upper UTI?

A) Suprapubic pain

B) Nausea and vomiting

C) Nocturia

D) Urinary incontinence

Answer: B) Nausea and vomiting

8. Which of the following is a less common symptom of UTI in elderly patients?

A) Altered mental status

B) Dysuria

C) Cloudy urine

D) Frequency

Answer: A) Altered mental status

9. Recurrent UTIs are generally defined as:

A) Two or more episodes of UTI in 12 months

B) Three or more episodes of UTI in 12 months

C) One UTI in 6 months

Printed with ChatGPT to PDF 25/95


D) One UTI per year

Answer: B) Three or more episodes of UTI in 12 months

10. Which of the following symptoms is most specific to a UTI?

A) Polyuria

B) Increased thirst

C) Dysuria and frequency

D) Nocturnal enuresis

Answer: C) Dysuria and frequency

Types of UTIs
11. An infection involving the bladder only is termed:

A) Urethritis

B) Cystitis

C) Pyelonephritis

D) Prostatitis

Answer: B) Cystitis

12. A UTI that ascends to infect the kidneys is called:

A) Pyelonephritis

B) Cystitis

C) Urethritis

D) Prostatitis

Answer: A) Pyelonephritis

13. Asymptomatic bacteriuria is typically treated in which of the following patient


populations?

A) Pregnant women

B) Non-pregnant women

C) Men without symptoms

Printed with ChatGPT to PDF 26/95


D) Children

Answer: A) Pregnant women

14. A UTI in a man is generally considered:

A) Complicated

B) Uncomplicated

C) Recurrent

D) Nosocomial

Answer: A) Complicated

15. The term “urosepsis” refers to:

A) A lower UTI with fever

B) Pyelonephritis with septicemia

C) Bacterial invasion limited to the bladder

D) Bacterial spread causing renal abscess

Answer: B) Pyelonephritis with septicemia

Diagnosis of UTIs
16. Which of the following tests is most commonly used for the initial diagnosis of a UTI?

A) Blood culture

B) Urinalysis with urine dipstick

C) Cystoscopy

D) Uroflowmetry

Answer: B) Urinalysis with urine dipstick

17. A positive urine dipstick test for leukocyte esterase indicates:

A) The presence of red blood cells in the urine

B) The presence of pus cells, indicating inflammation or infection

C) Alkaline urine

D) Glomerulonephritis

Printed with ChatGPT to PDF 27/95


Answer: B) The presence of pus cells, indicating inflammation or infection

18. What finding on urine culture is typically diagnostic of a UTI?

A) > 10,000 CFU/mL

B) > 100,000 CFU/mL

C) Presence of leukocytes only

D) Presence of epithelial cells

Answer: B) > 100,000 CFU/mL

19. In complicated UTIs, which imaging study is often recommended to evaluate for
structural abnormalities?

A) Abdominal X-ray

B) Renal ultrasound

C) MRI of the pelvis

D) Intravenous pyelogram (IVP)

Answer: B) Renal ultrasound

20. A patient presents with suspected pyelonephritis. Which laboratory test is most likely to
be abnormal?

A) Hemoglobin

B) White blood cell count

C) Liver function tests

D) Serum electrolytes

Answer: B) White blood cell count

Management of UTIs
21. The first-line antibiotic treatment for uncomplicated cystitis in women is:

A) Amoxicillin

B) Nitrofurantoin

C) Ciprofloxacin

Printed with ChatGPT to PDF 28/95


D) Vancomycin

Answer: B) Nitrofurantoin

22. Which antibiotic is recommended as first-line therapy for acute pyelonephritis?

A) Trimethoprim-sulfamethoxazole

B) Ciprofloxacin

C) Amoxicillin

D) Doxycycline

Answer: B) Ciprofloxacin

23. In cases of recurrent UTIs, which of the following strategies may reduce recurrence?

A) Avoiding cranberry juice

B) Prophylactic antibiotic therapy

C) Decreasing fluid intake

D) Delaying urination for longer periods

Answer: B) Prophylactic antibiotic therapy

24. A patient with catheter-associated UTI (CAUTI) is treated primarily by:

A) Administering antifungals

B) Removing or changing the catheter and initiating antibiotics

C) Increasing hydration only

D) Performing cystoscopy

Answer: B) Removing or changing the catheter and initiating antibiotics

25. Which of the following lifestyle modifications is commonly recommended to prevent UTIs
in women?

A) Reducing fluid intake

B) Delaying urination

C) Wiping from front to back after urination

D) Drinking more carbonated beverages

Printed with ChatGPT to PDF 29/95


Answer: C) Wiping from front to back after urination

26. In cases of pyelonephritis with septicemia, which of the following is the most appropriate
initial treatment?

A) Oral antibiotics

B) Intravenous broad-spectrum antibiotics

C) Hydration only

D) Observation without antibiotics

Answer: B) Intravenous broad-spectrum antibiotics

27. When should asymptomatic bacteriuria be treated with antibiotics?

A) In all elderly patients

B) In non-pregnant women with a history of recurrent UTI

C) In pregnant women

D) In patients with a history of kidney stones

Answer: C) In pregnant women

Urology Ward Care


28. In urology ward management, a patient with recurrent pyelonephritis and obstructive
uropathy may require:

A) Continuous antibiotic therapy

B) Urethral catheterization

C) Percutaneous nephrostomy

D) Bladder irrigation

Answer: C) Percutaneous nephrostomy

29. A young woman presents with her third UTI in six months. Which investigation is most
appropriate to evaluate the cause of recurrent infections?

A) Pelvic ultrasound

B) Urodynamic studies

Printed with ChatGPT to PDF 30/95


C) Renal function tests

D) Cystoscopy

Answer: A) Pelvic ultrasound

30. A male patient with UTI symptoms has persistent fever and perineal pain despite
antibiotic treatment. The next step should be:

A) Continue current antibiotics

B) Switch to a broader-spectrum antibiotic

C) Perform a urine culture

D) Evaluate for prostatitis with a digital rectal examination (DRE)

Answer: D) Evaluate for prostatitis with a digital rectal examination (DRE)

These questions cover the clinical spectrum of UTIs, including risk factors, symptoms,
diagnostic methods, types of UTIs, and management strategies. These BCQs are applicable
for both academic learning and practical understanding of UTI management in the urology
ward.

Here are 30 Best Choice Questions (BCQs) on renal neoplasms (causes, types, symptoms,
complications, diagnosis, and management) based on Bailey & Love’s Short Practice of Surgery
and urology ward practice.

Causes of Renal Neoplasms


1. Which of the following is the most common risk factor for renal cell carcinoma?

A) Hypertension

B) Smoking

C) Obesity

D) Chronic kidney disease

Answer: B) Smoking

2. Renal cell carcinoma has been associated with which of the following genetic syndromes?

Printed with ChatGPT to PDF 31/95


A) Von Hippel-Lindau syndrome

B) Marfan syndrome

C) Down syndrome

D) Cushing’s syndrome

Answer: A) Von Hippel-Lindau syndrome

3. Long-term use of which of the following medications is a known risk factor for renal cell
carcinoma?

A) NSAIDs

B) ACE inhibitors

C) Oral contraceptives

D) Proton pump inhibitors

Answer: A) NSAIDs

4. Which of the following is a modifiable risk factor for renal cancer?

A) Genetic mutations

B) Age over 50 years

C) Cigarette smoking

D) Male gender

Answer: C) Cigarette smoking

5. Chronic exposure to which chemical is a known occupational risk factor for renal cancer?

A) Benzene

B) Cadmium

C) Asbestos

D) Lead

Answer: B) Cadmium

Types of Renal Neoplasms


6. The most common type of renal neoplasm is:

Printed with ChatGPT to PDF 32/95


A) Renal cell carcinoma

B) Transitional cell carcinoma

C) Wilms’ tumor

D) Oncocytoma

Answer: A) Renal cell carcinoma

7. Transitional cell carcinoma commonly arises from which part of the urinary tract?

A) Renal cortex

B) Ureter and renal pelvis

C) Renal medulla

D) Proximal tubule

Answer: B) Ureter and renal pelvis

8. Wilms' tumor is primarily a renal neoplasm seen in:

A) Elderly males

B) Middle-aged women

C) Children

D) Pregnant women

Answer: C) Children

9. A benign renal tumor characterized by the presence of large eosinophilic cells is:

A) Angiomyolipoma

B) Oncocytoma

C) Papillary carcinoma

D) Clear cell carcinoma

Answer: B) Oncocytoma

10. Which of the following renal neoplasms is commonly associated with tuberous sclerosis?

A) Angiomyolipoma

B) Renal cell carcinoma

Printed with ChatGPT to PDF 33/95


C) Chromophobe carcinoma

D) Wilms' tumor

Answer: A) Angiomyolipoma

Symptoms of Renal Neoplasms


11. The classical triad of symptoms in renal cell carcinoma includes:

A) Flank pain, hematuria, and a palpable mass

B) Dysuria, nocturia, and fever

C) Hematuria, urinary retention, and weight loss

D) Flank pain, urinary frequency, and polyuria

Answer: A) Flank pain, hematuria, and a palpable mass

12. A common symptom of a large renal neoplasm that causes renal vein thrombosis is:

A) Varicocele

B) Edema of the lower limbs

C) Hematuria

D) Flank pain

Answer: A) Varicocele

13. Which of the following is a common paraneoplastic syndrome associated with renal cell
carcinoma?

A) Hyperthyroidism

B) Hypercalcemia

C) Hypokalemia

D) Hypoglycemia

Answer: B) Hypercalcemia

14. Weight loss and night sweats are systemic symptoms most often seen in which type of
renal neoplasm?

A) Transitional cell carcinoma

Printed with ChatGPT to PDF 34/95


B) Wilms' tumor

C) Renal cell carcinoma

D) Oncocytoma

Answer: C) Renal cell carcinoma

15. A patient with a history of hematuria and flank pain presents with sudden onset of a left-
sided varicocele. This finding is most suspicious for:

A) Testicular torsion

B) Renal vein thrombosis

C) Renal cell carcinoma

D) Bladder cancer

Answer: C) Renal cell carcinoma

Complications of Renal Neoplasms


16. The most common site of metastasis for renal cell carcinoma is:

A) Lungs

B) Liver

C) Bone

D) Brain

Answer: A) Lungs

17. Renal cell carcinoma commonly invades which of the following structures?

A) Renal pelvis

B) Renal vein

C) Renal artery

D) Ureter

Answer: B) Renal vein

18. Which of the following complications can arise from a large renal neoplasm?

A) Hypertension due to compression of the renal artery

Printed with ChatGPT to PDF 35/95


B) Renal stone formation

C) Renal abscess

D) Urethral stricture

Answer: A) Hypertension due to compression of the renal artery

19. Wilms’ tumor is commonly associated with which congenital syndrome?

A) Beckwith-Wiedemann syndrome

B) Klinefelter syndrome

C) Down syndrome

D) Turner syndrome

Answer: A) Beckwith-Wiedemann syndrome

20. A renal neoplasm leading to compression of the ureter is likely to cause:

A) Hydronephrosis

B) Pyelonephritis

C) Glomerulonephritis

D) Ureteral rupture

Answer: A) Hydronephrosis

Diagnosis of Renal Neoplasms


21. Which imaging modality is the most sensitive for detecting renal masses?

A) Intravenous pyelogram (IVP)

B) Renal ultrasound

C) Computed tomography (CT) scan

D) Magnetic resonance imaging (MRI)

Answer: C) Computed tomography (CT) scan

22. In diagnosing renal cell carcinoma, which of the following tests is commonly used to
evaluate metastasis?

A) Chest X-ray

Printed with ChatGPT to PDF 36/95


B) Bone scan

C) PET scan

D) Renal function test

Answer: B) Bone scan

23. Which laboratory finding may be present in a patient with renal cell carcinoma?

A) Hypocalcemia

B) Polycythemia

C) Hypoglycemia

D) Hyperkalemia

Answer: B) Polycythemia

24. The definitive diagnosis of a renal mass is made by:

A) Needle biopsy

B) Fine needle aspiration (FNA)

C) Histopathological examination after surgical resection

D) Urine cytology

Answer: C) Histopathological examination after surgical resection

25. What is the most appropriate initial investigation for a suspected Wilms’ tumor in a child?

A) Intravenous pyelogram

B) CT abdomen

C) Renal ultrasound

D) MRI abdomen

Answer: C) Renal ultrasound

Management of Renal Neoplasms


26. The primary treatment for localized renal cell carcinoma is:

A) Chemotherapy

B) Radical nephrectomy

Printed with ChatGPT to PDF 37/95


C) Radiofrequency ablation

D) Partial nephrectomy

Answer: B) Radical nephrectomy

27. In patients with metastatic renal cell carcinoma, which of the following is often used as a
systemic therapy?

A) Immunotherapy with checkpoint inhibitors

B) Radiotherapy

C) Hormonal therapy

D) Antibiotics

Answer: A) Immunotherapy with checkpoint inhibitors

28. The treatment of choice for a small renal mass (<4 cm) in patients with a single kidney is:

A) Radical nephrectomy

B) Partial nephrectomy

C) Chemotherapy

D) Observation

Answer: B) Partial nephrectomy

29. In managing Wilms’ tumor, the treatment protocol typically includes:

A) Surgery followed by chemotherapy

B) Radiation therapy only

C) Observation and follow-up

D) Chemotherapy followed by surgery

Answer: A) Surgery followed by chemotherapy

30. Which of the following is used for the palliative management of bone metastases in renal
cell carcinoma?

A) Chemotherapy

B) Bisphosphonates or denosumab

Printed with ChatGPT to PDF 38/95


C) Angiographic embolization

D) Radical nephrectomy

Answer: B) Bisphosphonates or denosumab

These questions provide comprehensive coverage of renal neoplasms, including risk factors,
clinical features, diagnosis, complications, and treatment options. They are designed for both
academic and clinical application in the urology ward context.

Here are 30 scenario-based Best Choice Questions (BCQs) covering urolithiasis, benign
prostatic hyperplasia (BPH), prostate cancer, urinary tract infections (UTIs), and renal neoplasms
based on Bailey & Love's Short Practice of Surgery.

Urolithiasis
1. A 45-year-old male presents to the emergency department with severe, colicky right flank
pain radiating to the groin. He has gross hematuria and nausea. A non-contrast CT scan
reveals a 6 mm stone in the proximal ureter. What is the next best step in management?

A) Percutaneous nephrolithotomy

B) Ureteroscopy with stone removal

C) Observation and medical therapy

D) Extracorporeal shockwave lithotripsy (ESWL)

Answer: C) Observation and medical therapy


(Most ureteral stones ≤6 mm pass spontaneously with hydration and analgesia.)

2. A patient with a history of recurrent calcium oxalate kidney stones is found to have
hypercalciuria on 24-hour urine collection. What dietary advice would be most appropriate to
prevent stone recurrence?

A) Decrease dietary calcium

B) Increase dietary oxalate

C) Increase fluid intake and reduce salt intake

D) Increase protein intake

Answer: C) Increase fluid intake and reduce salt intake


(Increased fluids and reducing sodium intake decrease calcium excretion.)

Printed with ChatGPT to PDF 39/95


3. A 33-year-old woman presents with sudden-onset left-sided flank pain. She also reports
fever and dysuria. A CT scan reveals a 4 mm stone in the left ureter, and urinalysis shows
bacteria. What is the most appropriate next step in management?

A) ESWL

B) Antibiotics and hydration

C) Immediate surgery

D) Ureteral stent placement

Answer: B) Antibiotics and hydration


(Infected stones require antibiotic treatment and may pass spontaneously.)

4. A patient with a history of urolithiasis presents with a staghorn calculus. What type of
stone is most likely responsible for this finding?

A) Uric acid stone

B) Calcium oxalate stone

C) Struvite stone

D) Cystine stone

Answer: C) Struvite stone


(Struvite stones are commonly associated with infections and form staghorn calculi.)

5. A 65-year-old male presents with recurrent kidney stones and severe hypercalcemia. He
has a history of primary hyperparathyroidism. What is the most definitive management for
his condition?

A) Increase fluid intake and dietary modifications

B) ESWL for future stones

C) Surgical removal of parathyroid adenoma

D) Long-term bisphosphonate therapy

Answer: C) Surgical removal of parathyroid adenoma


(Correcting the cause of hypercalcemia prevents future stone formation.)

BPH

Printed with ChatGPT to PDF 40/95


6. A 70-year-old man complains of urinary frequency, nocturia, and a weak urinary stream.
Digital rectal examination reveals a symmetrically enlarged prostate. What is the first-line
treatment for this patient?

A) Finasteride

B) Tamsulosin

C) TURP (transurethral resection of the prostate)

D) Observation only

Answer: B) Tamsulosin
(Alpha-blockers are the first-line therapy for symptomatic BPH.)

7. A 65-year-old male with BPH presents with acute urinary retention. A Foley catheter is
placed, and 800 mL of urine is drained. What is the next step in management?

A) Immediate TURP

B) Trial of alpha-blocker therapy

C) Chronic indwelling catheter

D) Antibiotics and observation

Answer: B) Trial of alpha-blocker therapy


(Alpha-blockers help relieve obstruction, and a trial without catheter can be attempted after
medical management.)

8. A patient with BPH is not responding to medical therapy and experiences recurrent urinary
tract infections due to incomplete bladder emptying. What is the best next step in
management?

A) Increase the dose of alpha-blockers

B) TURP

C) Intermittent catheterization

D) Watchful waiting

Answer: B) TURP
(Surgical intervention is indicated in patients with BPH who have recurrent UTIs or bladder
stones.)

Printed with ChatGPT to PDF 41/95


9. A patient with known BPH undergoes a PSA test. The PSA level is elevated. Which factor
can cause a false elevation of PSA?

A) Recent ejaculation

B) Smoking

C) Blood pressure medication

D) Diabetes

Answer: A) Recent ejaculation


(PSA can be elevated after ejaculation, urinary retention, or prostate manipulation.)

10. A 72-year-old male with chronic BPH presents with nocturnal enuresis and overflow
incontinence. Post-void residual volume is elevated on bladder ultrasound. What
complication of BPH is he likely experiencing?

A) Urethral stricture

B) Bladder diverticula

C) Bladder decompensation

D) Renal failure

Answer: C) Bladder decompensation


(Chronic obstruction can cause detrusor muscle failure, leading to overflow incontinence.)

Prostate Cancer
11. A 68-year-old male presents with difficulty urinating and back pain. His PSA level is 15
ng/mL. Bone scans show multiple areas of increased uptake. What is the most likely
diagnosis?

A) BPH with osteoarthritis

B) Prostate cancer with bone metastasis

C) Bladder cancer with bone metastasis

D) Renal cell carcinoma

Answer: B) Prostate cancer with bone metastasis


(Bone metastasis is common in advanced prostate cancer.)

Printed with ChatGPT to PDF 42/95


12. A 60-year-old man is diagnosed with localized prostate cancer. What is the best treatment
option for a patient with a life expectancy greater than 10 years?

A) Active surveillance

B) Radical prostatectomy

C) Androgen deprivation therapy

D) Palliative radiotherapy

Answer: B) Radical prostatectomy


(Surgery is recommended for localized cancer in younger or healthier patients.)

13. A patient with prostate cancer is found to have elevated serum alkaline phosphatase
levels. What does this suggest?

A) Hepatic metastasis

B) Osteoblastic bone metastasis

C) Pulmonary metastasis

D) Localized disease

Answer: B) Osteoblastic bone metastasis


(Elevated alkaline phosphatase indicates bone involvement in metastatic prostate cancer.)

14. A 58-year-old man with early-stage prostate cancer has chosen active surveillance. What
does active surveillance entail?

A) Periodic PSA testing and biopsies

B) Immediate radiation therapy

C) Hormonal therapy and watchful waiting

D) Prostatectomy after 1 year of observation

Answer: A) Periodic PSA testing and biopsies


(Active surveillance involves monitoring PSA levels and repeat biopsies to assess progression.)

15. A man with metastatic prostate cancer is started on androgen deprivation therapy. Which
of the following is a common side effect of this treatment?

A) Hypercalcemia

B) Increased libido

Printed with ChatGPT to PDF 43/95


C) Hot flashes

D) Renal failure

Answer: C) Hot flashes


(Androgen deprivation therapy often causes hot flashes due to hormonal changes.)

UTIs
16. A 35-year-old woman presents with dysuria, frequency, and urgency. Urinalysis reveals
leukocytes and nitrites. What is the most likely diagnosis?

A) Acute cystitis

B) Pyelonephritis

C) Urethritis

D) Interstitial cystitis

Answer: A) Acute cystitis


(Dysuria, frequency, and nitrites in urine are typical of uncomplicated cystitis.)

17. A 28-year-old pregnant woman presents with asymptomatic bacteriuria. What is the
appropriate management?

A) Observation

B) Antibiotics

C) Bladder catheterization

D) Urodynamic studies

Answer: B) Antibiotics
(Asymptomatic bacteriuria in pregnancy should be treated to prevent complications.)

18. A patient presents with fever, flank pain, and costovertebral angle tenderness. Urinalysis
shows WBC casts. What is the most likely diagnosis?

A) Cystitis

B) Acute pyelonephritis

C) Ureteral stone

D) Bladder cancer

Printed with ChatGPT to PDF 44/95


Answer: B) Acute pyelonephritis
(Fever, flank pain, and WBC casts are characteristic of upper UTI or pyelonephritis.)

19. A patient with recurrent UTIs undergoes imaging and is found to have vesicoureteral
reflux. What complication may result from this condition?

A) Renal scarring

B) Urethral stricture

C) Bladder carcinoma

D) Urethritis

Answer: A) Renal scarring


(Vesicoureteral reflux can lead to chronic pyelonephritis and renal scarring.)

20. A diabetic patient presents with

persistent UTIs despite multiple courses of antibiotics. Urine culture shows Escherichia coli.
What is the most appropriate next step in management?

A) Prophylactic antibiotics

B) Surgical removal of a bladder stone

C) Control of blood sugar and further imaging

D) Immediate cystoscopy

Answer: C) Control of blood sugar and further imaging


(Diabetics are at higher risk of complicated UTIs due to poor immune response, and glycemic
control is crucial.)

Renal Neoplasms
21. A 55-year-old man presents with painless hematuria, flank pain, and a palpable
abdominal mass. CT scan shows a 5 cm mass in the right kidney. What is the most likely
diagnosis?

A) Renal cell carcinoma

B) Transitional cell carcinoma

C) Angiomyolipoma

Printed with ChatGPT to PDF 45/95


D) Wilms' tumor

Answer: A) Renal cell carcinoma


(The triad of hematuria, flank pain, and abdominal mass is classic for renal cell carcinoma.)

22. A 60-year-old woman presents with hematuria and fatigue. CT scan reveals a large mass
in the left kidney with invasion of the renal vein. What is the next best step in management?

A) Nephrectomy

B) Chemotherapy

C) Radiation therapy

D) Observation

Answer: A) Nephrectomy
(Surgery is the primary treatment for renal cell carcinoma, even with venous invasion.)

23. A 5-year-old child presents with abdominal swelling and hematuria. A renal ultrasound
shows a large, heterogeneous mass. What is the most likely diagnosis?

A) Renal cell carcinoma

B) Wilms’ tumor

C) Neuroblastoma

D) Transitional cell carcinoma

Answer: B) Wilms' tumor


(Wilms’ tumor is the most common renal malignancy in children.)

24. A 63-year-old man is diagnosed with a small (<4 cm) renal cell carcinoma. He is
asymptomatic, and the tumor is localized. What is the most appropriate treatment?

A) Radical nephrectomy

B) Partial nephrectomy

C) Systemic chemotherapy

D) Active surveillance

Answer: B) Partial nephrectomy


(Partial nephrectomy is preferred for small, localized renal masses to preserve renal function.)

Printed with ChatGPT to PDF 46/95


25. A patient with a history of renal cell carcinoma presents with new-onset bone pain. Bone
scans show multiple osteoblastic lesions. What is the best management option?

A) Chemotherapy

B) Bisphosphonates or denosumab

C) Surgery for bone metastases

D) Observation only

Answer: B) Bisphosphonates or denosumab


(Bisphosphonates help in the management of bone metastases by preventing further bone
loss.)

26. A 70-year-old man with a history of smoking presents with painless gross hematuria. He
has no other symptoms. What is the most appropriate initial investigation?

A) Renal biopsy

B) Cystoscopy

C) Abdominal ultrasound

D) CT urography

Answer: D) CT urography
(CT urography is the preferred imaging modality to evaluate hematuria, particularly in
smokers at high risk of urothelial cancer.)

27. A 52-year-old male presents with hematuria and flank pain. Imaging reveals a renal mass
with necrosis. Biopsy confirms clear cell renal carcinoma. What is a key molecular change
associated with this type of tumor?

A) BRAF mutation

B) TP53 mutation

C) VHL gene inactivation

D) Rb gene deletion

Answer: C) VHL gene inactivation


(Inactivation of the VHL gene is commonly associated with clear cell renal carcinoma.)

Printed with ChatGPT to PDF 47/95


28. A patient with renal cell carcinoma undergoes surgery but is found to have lymph node
involvement. What is the prognosis in this case?

A) Excellent, with long-term survival expected

B) Good, with no need for further treatment

C) Poor, due to advanced disease

D) Intermediate, with a chance for disease control

Answer: C) Poor, due to advanced disease


(Lymph node involvement in renal cell carcinoma is associated with a poor prognosis.)

29. A 6-year-old boy presents with hematuria and an abdominal mass. He is diagnosed with
Wilms' tumor. What is the mainstay of treatment for this condition?

A) Nephrectomy followed by chemotherapy

B) Radiation therapy alone

C) Chemotherapy alone

D) Surveillance only

Answer: A) Nephrectomy followed by chemotherapy


(Wilms' tumor is treated with surgery followed by chemotherapy in most cases.)

30. A 55-year-old male is diagnosed with a renal mass and undergoes a nephrectomy.
Pathology confirms chromophobe renal cell carcinoma. What is a characteristic feature of
this tumor?

A) High metastatic potential

B) Associated with tuberous sclerosis

C) Favorable prognosis compared to other types of renal cancer

D) Associated with poor prognosis due to aggressive behavior

Answer: C) Favorable prognosis compared to other types of renal cancer


(Chromophobe renal cell carcinoma has a better prognosis than clear cell carcinoma.)

These scenario-based BCQs comprehensively cover clinical presentations, diagnostic


approaches, and management strategies for conditions such as urolithiasis, BPH, prostate

Printed with ChatGPT to PDF 48/95


cancer, UTIs, and renal neoplasms. These questions reflect real-life situations encountered in
urology wards and surgical practice based on Bailey & Love's Short Practice of Surgery.

Here are 30 scenario-based Best Choice Questions (BCQs) covering pediatric urology
conditions, including posterior urethral valves (PUV), pelviureteric junction obstruction (PUJO),
vesicoureteral reflux (VUR), and other pediatric urological conditions, based on Bailey & Love's
Short Practice of Surgery.

Posterior Urethral Valves (PUV)


1. A newborn male presents with a distended abdomen, poor urinary stream, and palpable
bladder. An ultrasound reveals bilateral hydronephrosis and a thick-walled bladder. What is
the most likely diagnosis?

A) Ureterocele

B) Posterior urethral valves (PUV)

C) Vesicoureteral reflux

D) Ureteropelvic junction obstruction

Answer: B) Posterior urethral valves (PUV)


(PUV is the most common cause of lower urinary tract obstruction in male infants, presenting
with a distended bladder and hydronephrosis.)

2. A 1-month-old male with PUV is undergoing initial management. What is the most
appropriate immediate treatment?

A) Foley catheter insertion to relieve bladder pressure

B) Immediate urethral valve ablation

C) Nephrectomy

D) Observation

Answer: A) Foley catheter insertion to relieve bladder pressure


(Bladder decompression with a catheter is the initial step in managing PUV.)

3. A 3-year-old boy with a history of PUV presents with recurrent UTIs and incontinence.
Despite previous valve ablation, he has persistent symptoms. What is the most likely cause?

A) Urethral stricture

Printed with ChatGPT to PDF 49/95


B) Renal dysplasia

C) Bladder dysfunction

D) Vesicoureteral reflux

Answer: C) Bladder dysfunction


(Children with PUV may develop bladder dysfunction due to chronic obstruction.)

4. What is the most definitive treatment for posterior urethral valves (PUV)?

A) Long-term catheterization

B) Urethral dilation

C) Urethral valve ablation

D) Circumcision

Answer: C) Urethral valve ablation


(Endoscopic ablation of the valves is the definitive treatment for PUV.)

5. A child with posterior urethral valves has chronic renal insufficiency due to prolonged
obstruction. What complication is he at risk of developing in the long term?

A) Hypertension

B) Hypocalcemia

C) Renal stones

D) Nephrotic syndrome

Answer: A) Hypertension
(Chronic renal insufficiency from PUV can lead to hypertension due to impaired renal
function.)

Pelviureteric Junction Obstruction (PUJO)


6. A 2-year-old male is evaluated for recurrent flank pain, particularly after fluid intake.
Ultrasound shows hydronephrosis, but no ureteral dilation. What is the most likely
diagnosis?

A) Vesicoureteral reflux

B) Posterior urethral valves

Printed with ChatGPT to PDF 50/95


C) Pelviureteric junction obstruction (PUJO)

D) Ureterocele

Answer: C) Pelviureteric junction obstruction (PUJO)


(PUJO presents as hydronephrosis without ureteral dilation and may cause pain after fluid
intake.)

7. A 6-month-old infant is diagnosed with PUJO after prenatal hydronephrosis was identified.
What is the appropriate management for asymptomatic cases?

A) Immediate surgery

B) Prophylactic antibiotics and observation

C) Nephrectomy

D) Ureteric reimplantation

Answer: B) Prophylactic antibiotics and observation


(In asymptomatic PUJO, observation with antibiotics may be appropriate while monitoring for
progression.)

8. A 7-year-old boy presents with recurrent episodes of flank pain. Imaging reveals significant
hydronephrosis with a dilated renal pelvis. Diuretic renography shows a delayed washout
phase. What is the best management option?

A) Observation only

B) Ureteral stent placement

C) Pyeloplasty

D) Nephrectomy

Answer: C) Pyeloplasty
(Pyeloplasty is the treatment of choice for significant PUJO with symptoms.)

9. What is the most common cause of pelviureteric junction obstruction (PUJO) in children?

A) Aberrant renal artery compressing the junction

B) Ureteral atresia

C) Ureteral valve dysfunction

D) Ureteral duplication

Printed with ChatGPT to PDF 51/95


Answer: A) Aberrant renal artery compressing the junction
(An aberrant vessel crossing the ureter is a common cause of PUJO.)

10. A patient with PUJO has worsening hydronephrosis and deteriorating renal function.
What is the best imaging modality to evaluate renal function in this scenario?

A) Ultrasound

B) Diuretic renography

C) Intravenous pyelogram (IVP)

D) CT scan

Answer: B) Diuretic renography


(Diuretic renography helps assess drainage and function in patients with PUJO.)

Vesicoureteral Reflux (VUR)


11. A 4-year-old girl presents with recurrent febrile UTIs. A voiding cystourethrogram (VCUG)
reveals grade III vesicoureteral reflux. What is the most appropriate initial management?

A) Surgical reimplantation of the ureter

B) Observation and prophylactic antibiotics

C) Immediate nephrectomy

D) Ureteral stent placement

Answer: B) Observation and prophylactic antibiotics


(Most cases of VUR are initially managed with prophylactic antibiotics, especially in lower
grades.)

12. Which of the following conditions is a known risk factor for developing vesicoureteral
reflux (VUR)?

A) Posterior urethral valves

B) Pelviureteric junction obstruction

C) Renal agenesis

D) Multicystic dysplastic kidney

Answer: A) Posterior urethral valves


(PUV can lead to secondary VUR due to increased bladder pressure.)

Printed with ChatGPT to PDF 52/95


13. A 3-year-old boy with VUR is undergoing follow-up. Renal ultrasound shows new renal
scarring. What is the best next step in management?

A) Continue prophylactic antibiotics

B) Perform ureteral reimplantation surgery

C) Place a Foley catheter

D) Watchful waiting

Answer: B) Perform ureteral reimplantation surgery


(Progressive renal scarring is an indication for surgical correction of VUR.)

14. A child with bilateral grade IV VUR presents with worsening renal function. What
complication is most likely contributing to this deterioration?

A) Hypertension

B) Renal calculi

C) Chronic pyelonephritis

D) Urinary incontinence

Answer: C) Chronic pyelonephritis


(Chronic pyelonephritis due to recurrent infections can lead to renal damage in high-grade
VUR.)

15. A 2-year-old boy is diagnosed with grade II VUR. What is the most likely outcome with
conservative management?

A) Spontaneous resolution as the child grows

B) Progression to grade V VUR

C) Development of hydronephrosis

D) Immediate need for surgery

Answer: A) Spontaneous resolution as the child grows


(Lower grades of VUR, such as grade II, often resolve spontaneously with growth.)

Other Pediatric Urological Conditions


16. A 5-year-old boy presents with swelling in the scrotum, which increases in size when he
cries and decreases when he is relaxed. What is the most likely diagnosis?

Printed with ChatGPT to PDF 53/95


A) Inguinal hernia

B) Hydrocele

C) Varicocele

D) Testicular torsion

Answer: A) Inguinal hernia


(An inguinal hernia presents with intermittent scrotal swelling, often visible when intra-
abdominal pressure increases.)

17. A 12-year-old boy complains of a painless mass above his left testicle. On physical
examination, the mass increases in size with Valsalva maneuver. What is the most likely
diagnosis?

A) Hydrocele

B) Varicocele

C) Epididymal cyst

D) Testicular torsion

Answer: B) Varicocele
(A varicocele is a dilatation of the veins of the pampiniform plexus and increases with
standing or Valsalva maneuver.)

18. A newborn male is diagnosed with bilateral cryptorchidism. What is the most appropriate
time for surgical intervention if the testes do not descend spontaneously?

A) Before 3 months of age

B) Between 6-12 months of age

C) Between 2-3 years of age

D) After 5 years of age

Answer: B) Between 6-12 months of age


(Orchiopexy is recommended between 6-12 months if the testes have not descended
spontaneously.)

19. A 9-year-old boy presents with acute onset scrot

2/2

Printed with ChatGPT to PDF 54/95


al pain and swelling on the right side. He denies any trauma. The pain started suddenly while
he was playing. On examination, there is tenderness over the testicle, and the cremasteric
reflex is absent. What is the most likely diagnosis?

A) Testicular torsion

B) Epididymitis

C) Inguinal hernia

D) Hydrocele

Answer: A) Testicular torsion


(Testicular torsion presents with sudden onset scrotal pain, tenderness, and an absent
cremasteric reflex, requiring urgent intervention.)

20. A 4-year-old boy presents with a painless swelling in the groin, which extends to the
scrotum. The swelling transilluminates. What is the most likely diagnosis?

A) Hydrocele

B) Inguinal hernia

C) Testicular torsion

D) Varicocele

Answer: A) Hydrocele
(A hydrocele presents as a painless scrotal swelling that transilluminates, and it is typically
due to a collection of fluid around the testicle.)

21. A 5-year-old boy presents with bilateral undescended testes. An ultrasound confirms
intra-abdominal testes. What is the next step in management?

A) Hormonal therapy

B) Watchful waiting until puberty

C) Bilateral orchiopexy

D) Observation and regular follow-up

Answer: C) Bilateral orchiopexy


(Orchiopexy is recommended for cryptorchidism, especially if the testes are not in the scrotum
by the age of 1 year.)

Printed with ChatGPT to PDF 55/95


22. A 10-year-old boy presents with a persistent urinary dribbling despite being toilet trained.
A renal ultrasound reveals a dilated posterior urethra and a trabeculated bladder. What is the
most likely cause?

A) Urethral stricture

B) Vesicoureteral reflux

C) Posterior urethral valves

D) Neurogenic bladder

Answer: C) Posterior urethral valves


(PUV can lead to urinary dribbling, a trabeculated bladder, and a dilated posterior urethra.)

23. A 7-year-old boy is evaluated for enuresis and recurrent UTIs. A VCUG reveals grade IV
vesicoureteral reflux. What is the next step in management?

A) Prophylactic antibiotics and close monitoring

B) Immediate surgical correction

C) Ureteric stent placement

D) Long-term catheterization

Answer: A) Prophylactic antibiotics and close monitoring


(In cases of high-grade VUR, prophylactic antibiotics with regular follow-up is often the initial
step unless complications develop.)

24. A newborn male has antenatally detected hydronephrosis. Postnatal ultrasound confirms
severe hydronephrosis with no visible ureter. What is the next step in diagnosis?

A) Cystoscopy

B) Diuretic renography

C) Voiding cystourethrogram (VCUG)

D) MRI

Answer: B) Diuretic renography


(Diuretic renography helps assess the function and drainage of the renal pelvis, particularly in
cases of suspected PUJO.)

Printed with ChatGPT to PDF 56/95


25. A 3-year-old girl with recurrent UTIs undergoes a voiding cystourethrogram (VCUG),
which shows vesicoureteral reflux. What is the grade of reflux if the urine is refluxing into the
ureter and renal pelvis with mild dilation?

A) Grade I

B) Grade II

C) Grade III

D) Grade IV

Answer: C) Grade III


(Grade III VUR involves reflux into the ureter and renal pelvis with mild to moderate dilation.)

26. A 4-year-old boy is referred for evaluation of a renal mass discovered during an
abdominal ultrasound. The child has been asymptomatic, with no history of trauma or
infection. What is the most appropriate next step in evaluation?

A) Biopsy of the mass

B) MRI of the abdomen

C) Voiding cystourethrogram (VCUG)

D) Serum tumor markers

Answer: B) MRI of the abdomen


(MRI can provide detailed information on renal masses in children, such as Wilms’ tumor or
other renal neoplasms.)

27. A 2-year-old boy presents with an abdominal mass and gross hematuria. Imaging reveals
a large renal mass consistent with Wilms' tumor. What is the most appropriate treatment?

A) Nephrectomy followed by chemotherapy

B) Observation and repeat imaging

C) Chemotherapy alone

D) Radiation therapy

Answer: A) Nephrectomy followed by chemotherapy


(Nephrectomy is the primary treatment for Wilms' tumor, followed by chemotherapy to
prevent metastasis.)

Printed with ChatGPT to PDF 57/95


28. A 5-year-old girl presents with intermittent abdominal pain and vomiting. An ultrasound
reveals severe left-sided hydronephrosis, and a diuretic renogram confirms pelviureteric
junction obstruction (PUJO). What is the definitive treatment?

A) Ureteral stent placement

B) Nephrectomy

C) Pyeloplasty

D) Observation only

Answer: C) Pyeloplasty
(Pyeloplasty is the definitive treatment for PUJO to relieve the obstruction and preserve renal
function.)

29. A 7-year-old boy presents with a painless scrotal swelling that has been gradually
increasing in size. On examination, it does not transilluminate, and a “bag of worms”
appearance is noted. What is the most likely diagnosis?

A) Varicocele

B) Hydrocele

C) Inguinal hernia

D) Epididymal cyst

Answer: A) Varicocele
(A varicocele presents as a non-transilluminating scrotal swelling with a “bag of worms”
appearance, typically affecting the left side.)

30. A 3-year-old girl presents with recurrent UTIs and daytime incontinence. A VCUG shows
grade IV vesicoureteral reflux. Despite medical management, she develops progressive renal
scarring. What is the next best step in management?

A) Continue antibiotics and observation

B) Ureteral reimplantation surgery

C) Ureteral stenting

D) Bladder augmentation

Answer: B) Ureteral reimplantation surgery


(Surgical correction, such as ureteral reimplantation, is indicated in high-grade VUR with

Printed with ChatGPT to PDF 58/95


progressive renal damage.)

These scenario-based BCQs cover key pediatric urology conditions, emphasizing clinical
presentation, diagnosis, and management as seen in real-world practice. They are based on
guidelines and principles found in Bailey & Love's Short Practice of Surgery.

Here are 20 scenario-based Best Choice Questions (BCQs) on renal and ureteric trauma,
focusing on causes, types, complications, and management, based on Bailey & Love's Short
Practice of Surgery and relevant urology principles.

Renal Trauma
1. A 24-year-old male presents with flank pain and hematuria after a motor vehicle accident.
A CT scan reveals a renal contusion without urinary extravasation. What is the most
appropriate initial management?

A) Immediate nephrectomy

B) Observation with serial hematocrits

C) Exploratory laparotomy

D) Ureteral stenting

Answer: B) Observation with serial hematocrits


(Renal contusions without urinary extravasation are usually managed conservatively with
observation and monitoring for complications.)

2. A patient presents with gross hematuria following a fall from a height. CT imaging shows a
shattered kidney with extravasation of urine into the perirenal space. What is the likely grade
of renal injury?

A) Grade I

B) Grade III

C) Grade IV

D) Grade V

Answer: D) Grade V
(Grade V renal injury involves a shattered kidney, often with urinary extravasation and
involvement of the renal vasculature.)

Printed with ChatGPT to PDF 59/95


3. A 30-year-old male presents after blunt trauma to the abdomen during a sports injury. He
has visible hematuria. What is the most appropriate imaging modality for assessing the
extent of renal injury?

A) Ultrasound

B) CT scan with contrast

C) Intravenous pyelogram (IVP)

D) Plain abdominal X-ray

Answer: B) CT scan with contrast


(CT with contrast is the gold standard imaging for evaluating renal trauma, providing detailed
information on the extent of injury.)

4. A patient with a known renal injury following a car accident develops a sudden drop in
blood pressure, tachycardia, and flank swelling. What is the most likely complication?

A) Renal artery thrombosis

B) Renal abscess

C) Renal hematoma rupture

D) Urinary tract infection

Answer: C) Renal hematoma rupture


(A renal hematoma may rupture, leading to hemorrhage and hemodynamic instability.)

5. A 22-year-old presents with isolated renal trauma following a sports injury. There is a
minor laceration of the renal cortex on CT scan with minimal perirenal hematoma. What is
the most appropriate management?

A) Observation and bed rest

B) Immediate surgical exploration

C) Nephrectomy

D) Ureteral stenting

Answer: A) Observation and bed rest


(Minor renal injuries, such as small cortical lacerations, are typically managed conservatively
with observation.)

Printed with ChatGPT to PDF 60/95


6. A patient with blunt abdominal trauma has gross hematuria and severe left flank pain. CT
reveals a laceration extending into the renal pelvis with contrast extravasation. What is the
next best step in management?

A) Observation

B) Surgical repair

C) Nephrectomy

D) Angiographic embolization

Answer: B) Surgical repair


(Lacerations extending into the renal pelvis with urinary extravasation often require surgical
repair to prevent ongoing urine leakage and sepsis.)

7. A 35-year-old woman presents with flank pain after a bicycle accident. She has microscopic
hematuria, but her hemodynamic status is stable. What is the best approach to manage this
patient?

A) Discharge with pain management

B) Admission for observation and serial hematocrits

C) Immediate surgery

D) Foley catheter placement

Answer: B) Admission for observation and serial hematocrits


(In patients with renal trauma and microscopic hematuria but stable vitals, observation with
serial hematocrits is appropriate.)

8. Which of the following is the most common cause of renal trauma?

A) Penetrating injuries

B) Iatrogenic injury during surgery

C) Blunt trauma

D) Renal biopsy

Answer: C) Blunt trauma


(Blunt trauma, often from motor vehicle accidents, falls, or sports injuries, is the most
common cause of renal trauma.)

Printed with ChatGPT to PDF 61/95


9. A 28-year-old male sustains penetrating trauma to the flank. He is hemodynamically
unstable and has gross hematuria. What is the most appropriate initial step?

A) Immediate CT scan

B) Emergency nephrectomy

C) Intravenous fluids and emergency surgery

D) Conservative management

Answer: C) Intravenous fluids and emergency surgery


(In hemodynamically unstable patients with penetrating renal trauma, immediate
resuscitation and surgical exploration are necessary.)

10. A patient presents with flank pain and gross hematuria following a blunt abdominal
trauma. CT imaging shows a contained retroperitoneal hematoma without extravasation of
contrast. What is the best management approach?

A) Exploratory laparotomy

B) Embolization of renal artery

C) Conservative management with bed rest

D) Ureteral stenting

Answer: C) Conservative management with bed rest


(Stable retroperitoneal hematomas without active bleeding are typically managed
conservatively with observation.)

Ureteric Trauma
11. A 35-year-old woman undergoes an abdominal hysterectomy. Post-operatively, she
develops abdominal pain and leakage of urine from her incision site. What is the most likely
diagnosis?

A) Vesicovaginal fistula

B) Ureteric injury

C) Urinary tract infection

D) Bladder rupture

Printed with ChatGPT to PDF 62/95


Answer: B) Ureteric injury
(Ureteric injury is a known complication of pelvic surgeries such as hysterectomy and may
present with urinary leakage.)

12. A patient develops fever and flank pain three days after undergoing ureteroscopy for
stone removal. CT reveals urinoma formation around the left kidney. What is the most
appropriate management?

A) Immediate nephrectomy

B) Ureteral stent placement

C) Percutaneous drainage of the urinoma

D) Observation

Answer: C) Percutaneous drainage of the urinoma


(Urinomas are managed by drainage and ensuring proper urinary drainage through stenting
or repair.)

13. A patient is involved in a high-speed car accident and presents with flank ecchymosis and
hematuria. CT scan shows a transection of the left ureter. What is the most appropriate
management?

A) Immediate ureteral stenting

B) Observation and antibiotics

C) Ureteroureterostomy

D) Percutaneous nephrostomy

Answer: C) Ureteroureterostomy
(For ureteral transections, surgical repair such as ureteroureterostomy is the treatment of
choice.)

14. During a colectomy, a partial transection of the ureter is identified intraoperatively. What
is the most appropriate step?

A) Observation and post-operative imaging

B) Immediate ureteral repair

C) Ureteral stent placement

Printed with ChatGPT to PDF 63/95


D) Ureteric ligation and removal

Answer: B) Immediate ureteral repair


(If ureteral injury is identified during surgery, immediate repair is preferred to prevent
complications like urinoma and fistula formation.)

15. A patient develops flank pain and fever after abdominal trauma. CT scan reveals a
urinoma with extravasation of contrast. What is the next best step in management?

A) Foley catheter placement

B) Percutaneous nephrostomy

C) Observation and antibiotics

D) Immediate surgery

Answer: B) Percutaneous nephrostomy


(Percutaneous nephrostomy allows drainage of urine and resolution of the urinoma in the
setting of ureteric injury.)

16. A patient presents with hematuria following a pelvic fracture from a car accident. A CT
scan shows a high-grade ureteric injury with complete disruption. What is the most
appropriate initial step in management?

A) Surgical repair with ureteral reimplantation

B) Observation with antibiotics

C) Ureteral stent placement

D) Percutaneous nephrostomy

Answer: A) Surgical repair with ureteral reimplantation


(High-grade ureteric injuries often require surgical repair, such as ureteral reimplantation.)

17. A 28-year-old female has undergone a laparoscopic oophorectomy. Postoperatively, she


complains of abdominal pain and has minimal urine output despite adequate fluid intake. CT
urography shows hydronephrosis. What is the likely cause?

A) Ureteric ligation

B) Renal artery stenosis

C) Vesicovaginal fistula

Printed with ChatGPT to PDF 64/95


D) Urinary retention

Answer: A) Ureteric ligation


(Ureteric ligation during pelvic surgery can cause hydronephrosis and oliguria due to
obstruction.)

18. A patient with ureteral trauma presents with progressive flank pain and fever despite
ureteral stent placement. Imaging shows an enlarging perirenal collection. What is the next
best step?

A) Nephrectomy

B) Percutaneous drainage

B) Percutaneous drainage
(In the case of an enlarging perirenal collection, likely a urinoma or abscess, percutaneous
drainage is essential to relieve symptoms and prevent further complications.)

19. A 40-year-old male presents with gross hematuria and left-sided abdominal pain
following blunt trauma to the lower abdomen. A CT scan reveals extravasation of contrast
from the distal ureter without significant renal injury. What is the appropriate management?

A) Observation with antibiotics

B) Ureteral stent placement

C) Ureteroureterostomy

D) Nephrectomy

Answer: B) Ureteral stent placement


(For ureteral injuries with contrast extravasation, ureteral stenting is the preferred initial
management to allow for healing and prevent further leakage.)

20. A patient develops fever, abdominal distension, and flank pain one week after ureteric
trauma repair. Imaging reveals hydronephrosis and perinephric fluid collection. What is the
most likely complication?

A) Urinoma formation

B) Ureteral stricture

Printed with ChatGPT to PDF 65/95


C) Renal artery thrombosis

D) Pyelonephritis

Answer: A) Urinoma formation


(Urinomas are a common complication following ureteral injury or repair, resulting from
leakage of urine into the perinephric space.)

These 20 scenario-based BCQs cover various aspects of renal and ureteric trauma,
highlighting their causes, types of injuries, complications, and management strategies based
on Bailey & Love's Short Practice of Surgery.

Here are 30 Best Choice Questions (BCQs) covering benign testicular and scrotal conditions,
including Testicular Torsion, Epididymo-orchitis, Fournier's Gangrene, Varicocele, and Hydrocele,
based on Bailey & Love's Short Practice of Surgery and relevant urology guidelines.

Testicular Torsion
1. A 16-year-old boy presents with sudden onset of severe left testicular pain. On
examination, the left testis is tender, elevated, and the cremasteric reflex is absent. What is
the most likely diagnosis?

A) Testicular torsion

B) Epididymitis

C) Hydrocele

D) Varicocele

Answer: A) Testicular torsion


(Testicular torsion presents with acute pain, testicular elevation, and absence of the
cremasteric reflex, requiring urgent surgery.)

2. A 14-year-old boy presents with acute scrotal pain for 6 hours. On physical examination,
the left testis is swollen and tender, and there is no cremasteric reflex. What is the next best
step in management?

A) Ultrasound of the scrotum

B) Immediate surgical exploration

C) Antibiotic therapy

Printed with ChatGPT to PDF 66/95


D) Testicular biopsy

Answer: B) Immediate surgical exploration


(Testicular torsion is a surgical emergency, and exploration should not be delayed for
imaging.)

3. What is the most critical factor influencing the viability of the testis in testicular torsion?

A) Age of the patient

B) Duration of symptoms

C) Severity of pain

D) Bilaterality of torsion

Answer: B) Duration of symptoms


(The duration of symptoms is critical in determining testicular viability, with testicular salvage
rates decreasing rapidly after 6 hours.)

4. In testicular torsion, which of the following is typically absent?

A) Cremasteric reflex

B) Testicular swelling

C) Scrotal erythema

D) Abdominal pain

Answer: A) Cremasteric reflex


(The absence of the cremasteric reflex is a key clinical finding in testicular torsion.)

5. During surgical exploration for suspected testicular torsion, the testis is found to be viable.
What is the most appropriate management?

A) Detorsion with bilateral orchiopexy

B) Orchiectomy

C) Detorsion without fixation

D) Observation

Answer: A) Detorsion with bilateral orchiopexy


(Bilateral orchiopexy is recommended to prevent future torsion, as the contralateral testis is at
risk.)

Printed with ChatGPT to PDF 67/95


Epididymo-orchitis
6. A 32-year-old male presents with testicular pain and swelling for 3 days. He also reports
dysuria and fever. On examination, the epididymis is tender and swollen. What is the most
likely diagnosis?

A) Testicular torsion

B) Epididymo-orchitis

C) Varicocele

D) Testicular tumor

Answer: B) Epididymo-orchitis
(Epididymo-orchitis presents with gradual onset of scrotal pain, dysuria, and fever, and is
often associated with urinary tract infections.)

7. What is the most common bacterial pathogen responsible for epididymo-orchitis in


sexually active men under 35 years old?

A) Escherichia coli

B) Chlamydia trachomatis

C) Streptococcus species

D) Pseudomonas aeruginosa

Answer: B) Chlamydia trachomatis


(Chlamydia trachomatis is the most common cause of epididymo-orchitis in sexually active
men.)

8. A 45-year-old man presents with scrotal pain, fever, and dysuria. Examination reveals an
enlarged, tender epididymis and testis. He is diagnosed with epididymo-orchitis. What is the
most appropriate initial treatment?

A) Surgical exploration

B) Broad-spectrum antibiotics and scrotal elevation

C) NSAIDs and bed rest

D) Orchiectomy

Printed with ChatGPT to PDF 68/95


Answer: B) Broad-spectrum antibiotics and scrotal elevation
(Antibiotics targeting common uropathogens, along with supportive care like scrotal
elevation, are the standard treatment for epididymo-orchitis.)

9. Which of the following is a potential complication of untreated epididymo-orchitis?

A) Testicular torsion

B) Testicular infarction

C) Fournier’s gangrene

D) Hydrocele

Answer: B) Testicular infarction


(Untreated epididymo-orchitis can lead to testicular infarction due to compression of blood
flow.)

10. A patient presents with symptoms of epididymo-orchitis, but after initial treatment, his
symptoms do not resolve. What is the next best step?

A) Repeat antibiotics

B) Testicular ultrasound

C) Surgical exploration

D) CT scan of the abdomen

Answer: B) Testicular ultrasound


(If symptoms persist despite treatment, testicular ultrasound is indicated to rule out other
causes such as abscess or tumor.)

Fournier's Gangrene
11. A 60-year-old diabetic male presents with severe scrotal pain, swelling, and fever. His
scrotum is erythematous, with areas of necrosis. What is the most likely diagnosis?

A) Epididymo-orchitis

B) Testicular torsion

C) Fournier’s gangrene

D) Varicocele

Printed with ChatGPT to PDF 69/95


Answer: C) Fournier’s gangrene
(Fournier’s gangrene is a rapidly progressing necrotizing infection of the scrotum and
perineum, common in diabetics.)

12. What is the most appropriate initial management for Fournier’s gangrene?

A) Intravenous antibiotics and emergency debridement

B) Oral antibiotics and wound care

C) Scrotal elevation and observation

D) Hyperbaric oxygen therapy only

Answer: A) Intravenous antibiotics and emergency debridement


(Fournier’s gangrene is a surgical emergency requiring immediate antibiotics and extensive
debridement of necrotic tissue.)

13. What is a major risk factor for developing Fournier’s gangrene?

A) Young age

B) Immunosuppression

C) Varicocele

D) Recent scrotal surgery

Answer: B) Immunosuppression
(Immunosuppression, diabetes, and poor hygiene are major risk factors for developing
Fournier’s gangrene.)

14. A patient with Fournier’s gangrene has undergone initial surgical debridement. What is
the next step in their care?

A) Repeat debridement as needed

B) Orchiectomy

C) Hyperbaric oxygen therapy

D) Discharge with oral antibiotics

Answer: A) Repeat debridement as needed


(Patients with Fournier’s gangrene often require multiple debridements to fully control the
infection.)

Printed with ChatGPT to PDF 70/95


15. Which of the following imaging modalities is most useful for assessing the extent of soft
tissue infection in Fournier’s gangrene?

A) Ultrasound

B) CT scan

C) MRI

D) Plain X-ray

Answer: B) CT scan
(A CT scan can help assess the extent of necrotizing soft tissue infection and gas formation in
Fournier’s gangrene.)

Varicocele
16. A 22-year-old man presents with a painless scrotal swelling that becomes more
prominent when standing. Examination reveals a "bag of worms" appearance in the left
scrotum. What is the most likely diagnosis?

A) Hydrocele

B) Testicular torsion

C) Varicocele

D) Epididymitis

Answer: C) Varicocele
(Varicoceles present as a “bag of worms” appearance in the scrotum and are more prominent
when standing or straining.)

17. Which of the following is the most common side affected by varicocele?

A) Right side

B) Left side

C) Bilateral

D) Alternating sides

Answer: B) Left side


(Varicoceles most commonly affect the left side due to anatomical differences in venous
drainage.)

Printed with ChatGPT to PDF 71/95


18. A patient with varicocele is experiencing testicular atrophy and discomfort. What is the
best management option?

A) Observation

B) Scrotal support

C) Varicocelectomy

D) Oral antibiotics

Answer: C) Varicocelectomy
(Varicocelectomy is recommended in cases of testicular atrophy, discomfort, or infertility
associated with varicocele.)

19. Which of the following is a complication associated with untreated varicocele?

A) Testicular torsion

B) Infertility

C) Hydrocele

D) Fournier’s gangrene

Answer: B) Infertility
(Varicocele can lead to infertility due to impaired testicular function and abnormal sperm
parameters.)

**20

20. A 30-year-old male presents with a painless left scrotal swelling that decreases when
lying down. Examination confirms a varicocele. The patient is asymptomatic but worried
about fertility. What is the best approach?

A) Immediate surgery

B) Observation and reassurance

C) Oral medications for fertility

D) Scrotal elevation and antibiotics

Answer: B) Observation and reassurance


(In asymptomatic cases of varicocele without evidence of testicular atrophy or infertility,
observation and reassurance are appropriate.)

Printed with ChatGPT to PDF 72/95


Hydrocele
21. A 45-year-old male presents with a painless scrotal swelling that transilluminates on
examination. What is the most likely diagnosis?

A) Testicular torsion

B) Hydrocele

C) Epididymitis

D) Varicocele

Answer: B) Hydrocele
(A hydrocele presents as a painless scrotal swelling that transilluminates due to fluid
accumulation around the testicle.)

22. Which of the following is the best diagnostic test for confirming the presence of a
hydrocele?

A) CT scan

B) MRI

C) Testicular ultrasound

D) Doppler ultrasound

Answer: C) Testicular ultrasound


(Testicular ultrasound is the most useful test to confirm a hydrocele and rule out other scrotal
pathology.)

23. A patient with a small hydrocele is asymptomatic. What is the most appropriate
management?

A) Observation

B) Surgical excision

C) Needle aspiration

D) Antibiotic therapy

Answer: A) Observation
(Small, asymptomatic hydroceles are often managed with observation, as they may resolve
spontaneously.)

Printed with ChatGPT to PDF 73/95


24. What is the most common cause of hydrocele in adults?

A) Congenital anomaly

B) Inflammation or injury

C) Testicular tumor

D) Varicocele

Answer: B) Inflammation or injury


(Hydroceles in adults are most often secondary to inflammation, infection, trauma, or
surgery.)

25. A large hydrocele is causing discomfort to a 55-year-old male. What is the definitive
treatment?

A) Needle aspiration

B) Scrotal support

C) Hydrocelectomy

D) Observation

Answer: C) Hydrocelectomy
(Hydrocelectomy, the surgical excision of the hydrocele sac, is the definitive treatment for
symptomatic or large hydroceles.)

26. In which of the following conditions is a hydrocele most likely to complicate the clinical
course?

A) Varicocele

B) Testicular torsion

C) Epididymo-orchitis

D) Fournier’s gangrene

Answer: C) Epididymo-orchitis
(Hydroceles can develop secondary to inflammation or infection in the scrotal structures, such
as in epididymo-orchitis.)

Additional Conditions and Complications


27. Which of the following is NOT a common cause of benign scrotal swelling?

Printed with ChatGPT to PDF 74/95


A) Hydrocele

B) Varicocele

C) Testicular torsion

D) Testicular cancer

Answer: D) Testicular cancer


(Testicular cancer is a malignant condition, whereas hydrocele, varicocele, and torsion are
benign causes of scrotal swelling.)

28. A 50-year-old diabetic male develops a necrotizing infection of the perineum. What is the
most likely diagnosis?

A) Fournier’s gangrene

B) Epididymo-orchitis

C) Hydrocele

D) Testicular torsion

Answer: A) Fournier’s gangrene


(Fournier’s gangrene is a necrotizing fasciitis of the perineum and is a surgical emergency,
especially common in diabetic patients.)

29. Which of the following clinical signs is suggestive of Fournier’s gangrene?

A) Rapidly spreading scrotal swelling with crepitus

B) Gradual onset of testicular swelling

C) Painless scrotal mass that transilluminates

D) Scrotal pain relieved by elevation

Answer: A) Rapidly spreading scrotal swelling with crepitus


(Fournier’s gangrene is characterized by rapidly spreading infection with gas formation, often
palpable as crepitus.)

30. A patient presents with fever, scrotal pain, and a necrotic perineal lesion. The initial
management includes:

A) Observation

B) Intravenous antibiotics and emergency surgical debridement

Printed with ChatGPT to PDF 75/95


C) Scrotal support and oral antibiotics

D) Orchiectomy

Answer: B) Intravenous antibiotics and emergency surgical debridement


(Fournier’s gangrene requires urgent intravenous antibiotics and immediate surgical
debridement to remove necrotic tissue and control infection.)

These 30 Best Choice Questions (BCQs) cover benign testicular and scrotal conditions
including testicular torsion, epididymo-orchitis, Fournier's gangrene, varicocele, and hydrocele,
focusing on causes, clinical features, complications, and management strategies based on
Bailey & Love’s Short Practice of Surgery and relevant urological practices.

Here are 30 Best Choice Questions (BCQs) on Testicular Tumors covering causes, types, clinical
features, diagnosis, and management, based on Bailey & Love’s Short Practice of Surgery and
related urology practices.

Causes and Risk Factors


1. Which of the following is the most significant risk factor for developing testicular cancer?

A) Testicular torsion

B) Cryptorchidism

C) Varicocele

D) Hydrocele

Answer: B) Cryptorchidism
(Cryptorchidism, or undescended testis, is the most significant risk factor for developing
testicular cancer.)

2. What is the most common age range for the development of testicular cancer?

A) 10–20 years

B) 20–40 years

C) 40–60 years

Printed with ChatGPT to PDF 76/95


D) 60–80 years

Answer: B) 20–40 years


(Testicular cancer most commonly affects men between the ages of 20 and 40.)

3. Which of the following genetic conditions is associated with an increased risk of


developing testicular germ cell tumors?

A) Klinefelter syndrome

B) Turner syndrome

C) Down syndrome

D) Marfan syndrome

Answer: A) Klinefelter syndrome


(Klinefelter syndrome is associated with an increased risk of developing germ cell tumors.)

4. A man with a family history of testicular cancer is more likely to develop which type of
testicular tumor?

A) Seminoma

B) Leydig cell tumor

C) Yolk sac tumor

D) Sertoli cell tumor

Answer: A) Seminoma
(Family history is a risk factor for developing germ cell tumors, particularly seminomas.)

5. Which prenatal exposure is associated with an increased risk of testicular cancer in


adulthood?

A) Exposure to radiation

B) Use of diethylstilbestrol (DES)

C) Maternal smoking

D) High maternal protein intake

Answer: B) Use of diethylstilbestrol (DES)


(Maternal use of DES during pregnancy is associated with an increased risk of testicular
cancer in male offspring.)

Printed with ChatGPT to PDF 77/95


Types of Testicular Tumors
6. What is the most common type of testicular tumor?

A) Seminoma

B) Teratoma

C) Yolk sac tumor

D) Leydig cell tumor

Answer: A) Seminoma
(Seminoma is the most common type of testicular germ cell tumor.)

7. Which of the following testicular tumors is most commonly found in children?

A) Seminoma

B) Yolk sac tumor

C) Choriocarcinoma

D) Embryonal carcinoma

Answer: B) Yolk sac tumor


(Yolk sac tumors are the most common testicular tumors in children.)

8. A non-germ cell tumor that can cause precocious puberty due to increased androgen
production is:

A) Leydig cell tumor

B) Seminoma

C) Embryonal carcinoma

D) Sertoli cell tumor

Answer: A) Leydig cell tumor


(Leydig cell tumors can produce androgens, leading to precocious puberty in children.)

9. Which type of testicular tumor is associated with elevated alpha-fetoprotein (AFP) levels?

A) Seminoma

B) Choriocarcinoma

C) Yolk sac tumor

Printed with ChatGPT to PDF 78/95


D) Sertoli cell tumor

Answer: C) Yolk sac tumor


(Yolk sac tumors are associated with elevated AFP levels.)

10. What is the most aggressive type of testicular germ cell tumor, known for early
hematogenous spread?

A) Seminoma

B) Choriocarcinoma

C) Teratoma

D) Embryonal carcinoma

Answer: B) Choriocarcinoma
(Choriocarcinoma is an aggressive tumor with early hematogenous spread, especially to the
lungs.)

Clinical Features
11. The most common presenting symptom of a testicular tumor is:

A) Painful swelling of the scrotum

B) Painless testicular mass

C) Dysuria

D) Hematuria

Answer: B) Painless testicular mass


(A painless testicular mass is the most common presenting symptom of a testicular tumor.)

12. A patient with a testicular tumor presents with gynecomastia. Which type of testicular
tumor is most likely to cause this symptom?

A) Seminoma

B) Leydig cell tumor

C) Embryonal carcinoma

D) Teratoma

Printed with ChatGPT to PDF 79/95


Answer: B) Leydig cell tumor
(Leydig cell tumors can produce estrogen, leading to gynecomastia.)

13. Which of the following symptoms is an unusual but possible presenting sign of
metastatic testicular cancer?

A) Persistent cough

B) Scrotal swelling

C) Bone pain

D) Weight gain

Answer: C) Bone pain


(Bone pain can occur in advanced testicular cancer with skeletal metastases.)

14. Which of the following is a common site of metastasis for testicular germ cell tumors?

A) Lymph nodes

B) Brain

C) Pancreas

D) Skin

Answer: A) Lymph nodes


(Lymphatic spread, particularly to retroperitoneal lymph nodes, is common in testicular germ
cell tumors.)

15. A 28-year-old man presents with a painless mass in the left testis. What is the most
appropriate initial investigation?

A) Fine-needle aspiration (FNA)

B) Testicular biopsy

C) Ultrasound of the scrotum

D) CT scan of the abdomen

Answer: C) Ultrasound of the scrotum


(Scrotal ultrasound is the initial investigation of choice for evaluating a testicular mass.)

Diagnosis and Staging

Printed with ChatGPT to PDF 80/95


16. In testicular tumors, which tumor marker is most associated with seminomas?

A) Alpha-fetoprotein (AFP)

B) Human chorionic gonadotropin (hCG)

C) Lactate dehydrogenase (LDH)

D) Carcinoembryonic antigen (CEA)

Answer: B) Human chorionic gonadotropin (hCG)


(hCG may be elevated in seminomas, though it is more commonly associated with non-
seminomatous germ cell tumors.)

17. A man with testicular cancer undergoes radical orchiectomy. What is the next step in
staging the disease?

A) PET scan

B) Chest X-ray and abdominal CT scan

C) Bone marrow biopsy

D) Bone scan

Answer: B) Chest X-ray and abdominal CT scan


(Chest X-ray and CT of the abdomen and pelvis are used to assess for metastasis, especially to
lymph nodes and lungs.)

18. A patient has elevated AFP and hCG levels with a testicular mass on ultrasound. What is
the most likely diagnosis?

A) Seminoma

B) Mixed germ cell tumor

C) Leydig cell tumor

D) Sertoli cell tumor

Answer: B) Mixed germ cell tumor


(Elevated AFP and hCG suggest a mixed germ cell tumor, as pure seminomas do not elevate
AFP.)

19. Testicular tumor staging is primarily based on:

A) Tumor size

Printed with ChatGPT to PDF 81/95


B) Serum tumor markers

C) Lymph node involvement and distant metastasis

D) Age of onset

Answer: C) Lymph node involvement and distant metastasis


(Staging of testicular tumors focuses on lymph node involvement and the presence of distant
metastasis, typically assessed via CT scans and tumor marker levels.)

20. Which of the following tumor markers is useful in the follow-up of testicular cancer after
treatment?

A) CA-125

B) Prostate-specific antigen (PSA)

C) Lactate dehydrogenase (LDH)

D) AFP and hCG

Answer: D) AFP and hCG


(AFP and hCG are monitored during follow-up to detect recurrence of testicular cancer.)

Management
21. What is the standard treatment for a patient diagnosed with a localized testicular tumor?

A) Radiation therapy

B) Radical inguinal orchiectomy

C) Chemotherapy

D) Testicular biopsy

Answer: B) Radical inguinal orchiectomy


(Radical inguinal orchiectomy is the primary treatment for testicular cancer.)

22. Which of the following is the primary adjuvant treatment for seminoma after
orchiectomy in stage I disease?

A) Chemotherapy

B) Radiation therapy

C) Observation

Printed with ChatGPT to PDF 82/95


D) Retroperitoneal lymph node dissection

Answer: B) Radiation therapy


(Radiation therapy is often used as an adjuvant treatment for stage I seminomas following
orchiectomy.)

23. A patient with a non-seminomatous germ cell tumor (NSGCT) has elevated tumor
markers after orchiectomy. What is the

most appropriate next step in management?

A) Surveillance

B) Radiation therapy

C) Chemotherapy

D) Retroperitoneal lymph node dissection

Answer: C) Chemotherapy
(Chemotherapy is the standard approach for patients with NSGCT who have elevated tumor
markers or evidence of metastasis following orchiectomy.)

24. What is the chemotherapy regimen commonly used for metastatic non-seminomatous
germ cell tumors?

A) BEP (Bleomycin, Etoposide, Cisplatin)

B) ABVD (Adriamycin, Bleomycin, Vinblastine, Dacarbazine)

C) FOLFOX (Fluorouracil, Leucovorin, Oxaliplatin)

D) CHOP (Cyclophosphamide, Doxorubicin, Vincristine, Prednisone)

Answer: A) BEP (Bleomycin, Etoposide, Cisplatin)


(The BEP regimen is the standard chemotherapy protocol for metastatic non-seminomatous
germ cell tumors.)

25. Which treatment option is appropriate for advanced stage seminoma with
retroperitoneal lymph node involvement?

A) Surveillance

B) Radiotherapy to the retroperitoneum

C) High-dose chemotherapy with stem cell transplant

Printed with ChatGPT to PDF 83/95


D) Retroperitoneal lymph node dissection (RPLND)

Answer: B) Radiotherapy to the retroperitoneum


(Radiotherapy is effective in treating seminomas with retroperitoneal lymph node
involvement.)

26. Which of the following is a common side effect of Bleomycin used in the treatment of
testicular cancer?

A) Nephrotoxicity

B) Pulmonary toxicity

C) Hepatotoxicity

D) Neurotoxicity

Answer: B) Pulmonary toxicity


(Bleomycin is associated with pulmonary toxicity, particularly in patients receiving cumulative
doses.)

27. A 32-year-old patient undergoes a retroperitoneal lymph node dissection (RPLND) after
chemotherapy for a non-seminomatous germ cell tumor. What is the purpose of this
procedure?

A) Reduce the risk of recurrence

B) Detect residual or teratomatous tissue

C) Treat primary tumor

D) Relieve pain

Answer: B) Detect residual or teratomatous tissue


(RPLND is performed to detect and remove any residual tumor or teratoma following
chemotherapy in NSGCT patients.)

Prognosis and Follow-Up


28. After treatment for testicular cancer, tumor markers (AFP and hCG) are monitored
regularly. How often should markers be checked during the first year of follow-up?

A) Monthly

B) Every 3 months

Printed with ChatGPT to PDF 84/95


C) Every 6 months

D) Annually

Answer: B) Every 3 months


(Tumor markers are monitored every 3 months during the first year after treatment to detect
early recurrence.)

29. What is the long-term survival rate for patients with early-stage testicular cancer
following treatment?

A) 50%

B) 70%

C) 90%

D) Over 95%

Answer: D) Over 95%


(The long-term survival rate for patients with early-stage testicular cancer is excellent,
exceeding 95% with appropriate treatment.)

30. What is the most important factor determining the prognosis of testicular cancer?

A) Tumor size

B) Tumor markers and stage at diagnosis

C) Patient’s age

D) Presence of gynecomastia

Answer: B) Tumor markers and stage at diagnosis


(Tumor markers and the stage of the cancer at the time of diagnosis are the most significant
factors in determining prognosis.)

These 30 BCQs cover the essential aspects of testicular tumors, including causes, types,
clinical features, diagnosis, staging, management, and prognosis, based on Bailey & Love’s
Short Practice of Surgery and the standard clinical approach in urology wards.

Printed with ChatGPT to PDF 85/95


Here are 30 Best Choice Questions (BCQs) presented in scenario format, covering topics from
pediatric urology, renal and ureteric trauma, benign scrotal conditions, and testicular tumors
based on Bailey & Love’s Short Practice of Surgery.

Pediatric Urology
1. A 6-month-old male presents with recurrent urinary tract infections. An ultrasound shows
dilated renal pelvis and calyces on the left side. What is the most likely diagnosis?

A) Posterior urethral valves (PUV)

B) Vesicoureteral reflux (VUR)

C) Ureteropelvic junction obstruction (PUJO)

D) Neurogenic bladder

Answer: C) Ureteropelvic junction obstruction (PUJO)


(The presence of dilated renal pelvis and calyces suggests an obstruction at the ureteropelvic
junction.)

2. A newborn is diagnosed with posterior urethral valves after presenting with a distended
bladder and poor urinary stream. Which management is most appropriate?

A) Observation

B) Endoscopic valve ablation

C) Nephrectomy

D) Antibiotic prophylaxis

Answer: B) Endoscopic valve ablation


(Endoscopic ablation of the valves is the definitive treatment for posterior urethral valves.)

3. A 5-year-old girl presents with fever and flank pain. A renal ultrasound reveals
hydronephrosis. What is the most appropriate next step in management?

A) Urinary catheterization

B) Intravenous antibiotics

C) Voiding cystourethrogram (VCUG)

D) Ureteral stenting

Printed with ChatGPT to PDF 86/95


Answer: B) Intravenous antibiotics
(Given the fever and suspected infection, intravenous antibiotics should be initiated
immediately.)

4. An 8-year-old boy is evaluated for recurrent urinary tract infections and is found to have
vesicoureteral reflux (VUR). What is the most appropriate initial management?

A) Surgery

B) Antibiotic prophylaxis

C) Observation

D) Voiding cystourethrogram (VCUG)

Answer: B) Antibiotic prophylaxis


(Antibiotic prophylaxis is recommended to reduce the incidence of urinary tract infections in
children with VUR.)

5. A 3-year-old child presents with abdominal pain and has a palpable abdominal mass. An
ultrasound shows a large cystic mass in the kidney. What is the most likely diagnosis?

A) Wilms' tumor

B) Renal abscess

C) Hydronephrosis

D) Multicystic dysplastic kidney

Answer: D) Multicystic dysplastic kidney


(A cystic mass in a child is suggestive of multicystic dysplastic kidney.)

Renal and Ureteric Trauma


6. A 25-year-old male was involved in a road traffic accident and presents with hematuria and
flank pain. A CT scan shows a grade II renal injury. What is the best management option?

A) Nephrectomy

B) Observation

C) Ureteral stenting

D) Surgical exploration

Printed with ChatGPT to PDF 87/95


Answer: B) Observation
(Grade II renal injuries can typically be managed conservatively with observation and
supportive care.)

7. A patient presents with severe flank pain and hematoma formation after a sports injury.
What is the most appropriate imaging study to evaluate for renal injury?

A) X-ray abdomen

B) Ultrasound

C) CT scan with contrast

D) MRI

Answer: C) CT scan with contrast


(CT scan with contrast is the preferred imaging modality for evaluating renal trauma.)

8. Following a stab wound to the abdomen, a patient exhibits signs of hemorrhagic shock
and has blood at the urinary meatus. What is the most appropriate initial step?

A) Foley catheter insertion

B) CT scan of the abdomen

C) Retrograde urethrogram (RUG)

D) Urgent laparotomy

Answer: C) Retrograde urethrogram (RUG)


(Blood at the meatus necessitates a RUG to assess for urethral injury before catheter
placement.)

9. A patient with a right-sided ureteric injury due to a pelvic fracture is stable. What is the
best approach to manage this injury?

A) Immediate surgery

B) Ureteral stenting

C) Observation

D) Nephrectomy

Answer: B) Ureteral stenting


(Ureteral stenting can be effective for managing ureteric injuries, especially in stable patients.)

Printed with ChatGPT to PDF 88/95


10. A 30-year-old male presents with flank pain and has a laceration on the left side following
a fall. CT scan shows a laceration involving the renal pelvis. What is the most appropriate
management?

A) Conservative management

B) Surgical exploration and repair

C) Percutaneous nephrostomy

D) Nephrectomy

Answer: B) Surgical exploration and repair


(Involvement of the renal pelvis typically necessitates surgical repair to restore urinary
drainage.)

Benign Scrotal Conditions


11. A 12-year-old boy presents with a swollen left scrotum that is painful and has redness
over the skin. What is the most likely diagnosis?

A) Hydrocele

B) Testicular torsion

C) Epididymo-orchitis

D) Varicocele

Answer: C) Epididymo-orchitis
(Epididymo-orchitis is characterized by painful swelling and redness, often due to infection.)

12. A 30-year-old male complains of a dull ache in the scrotum and has a bag of worms-like
appearance on examination. What is the most likely diagnosis?

A) Testicular torsion

B) Hydrocele

C) Varicocele

D) Epididymitis

Answer: C) Varicocele
(A varicocele often presents as a "bag of worms" appearance due to dilated veins in the
scrotum.)

Printed with ChatGPT to PDF 89/95


13. A patient presents with a scrotal mass that transilluminates and is painless. What is the
most appropriate management for a hydrocele?

A) Immediate surgery

B) Observation

C) Needle aspiration

D) Antibiotic therapy

Answer: B) Observation
(Asymptomatic hydroceles can be managed with observation, as they may resolve
spontaneously.)

14. A 45-year-old man with diabetes presents with fever, scrotal pain, and erythema. What is
the most likely diagnosis?

A) Testicular torsion

B) Fournier’s gangrene

C) Epididymo-orchitis

D) Hydrocele

Answer: B) Fournier’s gangrene


(Fournier’s gangrene is a life-threatening necrotizing infection and is characterized by rapid
progression of symptoms.)

15. Which of the following is the most effective surgical treatment for symptomatic
hydrocele?

A) Needle aspiration

B) Scrotal support

C) Hydrocelectomy

D) Antibiotic therapy

Answer: C) Hydrocelectomy
(Hydrocelectomy is the definitive treatment for symptomatic hydrocele.)

Testicular Tumors

Printed with ChatGPT to PDF 90/95


16. A 28-year-old man presents with a painless mass in the right testis and elevated alpha-
fetoprotein (AFP) levels. What is the most likely diagnosis?

A) Seminoma

B) Embryonal carcinoma

C) Yolk sac tumor

D) Teratoma

Answer: C) Yolk sac tumor


(Elevated AFP levels are indicative of a yolk sac tumor.)

17. A patient presents with gynecomastia and a firm testicular mass. Ultrasound reveals a
Leydig cell tumor. What is the most appropriate management?

A) Observation

B) Radical orchiectomy

C) Chemotherapy

D) Radiation therapy

Answer: B) Radical orchiectomy


(Surgical removal of the tumor is the mainstay treatment for Leydig cell tumors.)

18. A 30-year-old male presents with scrotal swelling and an abdominal mass. What is the
most important initial investigation?

A) Serum tumor markers

B) Ultrasound of the scrotum

C) CT scan of the abdomen

D) MRI of the pelvis

Answer: B) Ultrasound of the scrotum


(Ultrasound is the first-line investigation for evaluating scrotal masses.)

19. A young man with a testicular tumor undergoes a radical orchiectomy. What is the next
best step in management?

A) Immediate chemotherapy

B) Regular follow-up with serum tumor markers

Printed with ChatGPT to PDF 91/95


C) Radiation therapy

D) No further treatment

Answer: B) Regular follow-up with serum tumor markers


(Monitoring serum tumor markers is essential after orchiectomy to detect any recurrence.)

20. A 25-year-old man presents with a firm, non-tender testicular mass and elevated human
chorionic gonadotropin (hCG). What type of testicular tumor is he most likely to

have?

A) Seminoma

B) Non-seminomatous germ cell tumor

C) Sertoli cell tumor

D) Leydig cell tumor

Answer: B) Non-seminomatous germ cell tumor


(Elevated hCG is commonly associated with non-seminomatous germ cell tumors, particularly
choriocarcinoma or mixed germ cell tumors.)

Combined Scenarios
21. A 4-year-old boy with a history of recurrent UTIs is found to have posterior urethral
valves. What is the most common long-term complication associated with this condition?

A) Chronic kidney disease

B) Urinary incontinence

C) Recurrent UTIs

D) Vesicoureteral reflux

Answer: A) Chronic kidney disease


(Posterior urethral valves can lead to significant bladder outlet obstruction, resulting in long-
term renal damage and chronic kidney disease.)

22. A 20-year-old male presents with sudden onset of severe scrotal pain, and a Doppler
ultrasound shows absent blood flow to the testis. What is the most likely diagnosis?

A) Epididymitis

Printed with ChatGPT to PDF 92/95


B) Testicular torsion

C) Testicular rupture

D) Varicocele

Answer: B) Testicular torsion


(The absence of blood flow on Doppler ultrasound is indicative of testicular torsion, a surgical
emergency.)

23. A 10-year-old boy is diagnosed with a Wilms' tumor. What is the most important pre-
operative consideration?

A) Chemotherapy

B) Ultrasound follow-up

C) Renal function assessment

D) Imaging of the lungs

Answer: C) Renal function assessment


(Assessment of renal function is crucial before surgery for Wilms' tumor due to the potential
for associated renal dysfunction.)

24. A 60-year-old man with a history of smoking presents with hematuria and a mass on
imaging. What is the most likely diagnosis?

A) Testicular cancer

B) Renal cell carcinoma

C) Benign prostatic hyperplasia

D) Transitional cell carcinoma

Answer: B) Renal cell carcinoma


(Hematuria and a renal mass in a smoker raise suspicion for renal cell carcinoma.)

25. A young athlete experiences a renal contusion during a football match. What is the best
initial management for a stable patient with a renal injury?

A) Immediate nephrectomy

B) Observation and bed rest

C) Urgent renal angiography

Printed with ChatGPT to PDF 93/95


D) Intravenous hydration

Answer: B) Observation and bed rest


(Stable renal injuries can be managed conservatively with observation and bed rest.)

26. A 5-year-old boy is evaluated for a scrotal mass, and an ultrasound reveals a solid mass
with calcifications. What is the most likely diagnosis?

A) Teratoma

B) Epididymitis

C) Hydrocele

D) Varicocele

Answer: A) Teratoma
(A solid testicular mass with calcifications in a child is suggestive of a teratoma.)

27. A patient with a known history of testicular cancer presents for follow-up with elevated
serum tumor markers. What is the most appropriate next step?

A) Surgery

B) Repeat imaging

C) Initiate chemotherapy

D) Observe without intervention

Answer: B) Repeat imaging


(Elevated tumor markers necessitate further imaging to assess for recurrence or metastasis.)

28. A 35-year-old male presents with a sudden onset of pain in the scrotum. Physical
examination reveals a high-riding testis. What is the most appropriate management?

A) Ice application

B) Surgical detorsion and fixation

C) Antibiotics

D) Observation

Answer: B) Surgical detorsion and fixation


(High-riding testis and sudden pain suggest testicular torsion, which requires urgent surgical
intervention.)

Printed with ChatGPT to PDF 94/95


29. A 29-year-old man presents with a scrotal mass, which is firm and does not
transilluminate. What is the most appropriate initial investigation?

A) Serum tumor markers

B) CT scan of the abdomen

C) Scrotal ultrasound

D) MRI of the pelvis

Answer: C) Scrotal ultrasound


(Ultrasound is the first-line investigation to characterize a scrotal mass.)

30. A 32-year-old man with a recent diagnosis of a seminoma is being counseled about the
prognosis. What is the typical 5-year survival rate for stage I seminoma after treatment?

A) 50%

B) 75%

C) 90%

D) Over 95%

Answer: D) Over 95%


(Stage I seminoma has an excellent prognosis, with a 5-year survival rate exceeding 95%
following treatment.)

These 30 BCQs cover a variety of scenarios from pediatric urology, renal and ureteric trauma,
benign scrotal conditions, and testicular tumors, providing comprehensive questions and
answers based on the guidelines and knowledge presented in Bailey & Love’s Short Practice of
Surgery.

Printed with ChatGPT to PDF 95/95

You might also like